Pediatric Flashcards

1
Q

(Chest) 8. Which of the following features is more likely to suggest extralobar rather than intralobar sequestration?

A. Visceral pleural investment

B. Greater association with congenital abnormalities

C. Repeated infections

D. Pulmonary venous drainage

E. Presentation in adulthood

A

B. Greater association with congenital abnormalities

The association with congenital abnormalities is greater in extralobar (50%) than intralobar sequestration (15%). Other features of extralobar sequestration include systemic venous drainage, having its own plural investment, and being symptomatic in the first 6 months of life.

How well did you know this?
1
Not at all
2
3
4
5
Perfectly
2
Q

(Chest) 18. A 40-year-old man with recurrent left lower lobe pneumonias presents with fever and cough. CT shows a well-defined area of low attenuation with fine enhancing septae within the area of consolidation. An anomalous vessel supplies the mass, apparently arising directly from the aorta. What is the most likely diagnosis?

A. Intralobar sequestration

B. Lymphoma

C. Bronchoalvolar cell carcinoma

D. Infected bronchogenic cyst

E. Congenital cystic adenomatoid malformation

A

A. Intralobar sequestration

The presence of systemic vascular supply to the infected segment and the history of recurrent infections makesintralobar sequestration the most likely diagnosis.

How well did you know this?
1
Not at all
2
3
4
5
Perfectly
3
Q
  1. Which is the most common location for congenital lobar hyperinflation?

A. Left Upper Lobe (LUL)

B. Right Upper Lobe (RUL)

C. Right Middle Lobe (RML)

D. Right Lower Lobe (RLL)

E. Left Lower lobe (LLL)

A

A. Left Upper Lobe (LUL)

Congenital lobar hyperinflation occurs most commonly in the LUL (42%) followed by the RML (35%). Imaging features include a radiodense lobe that becomes radiolucent and hyperexpanded as fetal fluid is replaced with air.

How well did you know this?
1
Not at all
2
3
4
5
Perfectly
4
Q
  1. As part of investigations for respiratory distress in a neonate, a CT chest is performed showing variable sized cysts in the left lower lobe which are measuring 3cm (multicystic mass of pulmonary tissue). Regarding congenital pulmonary airway malformation, which of the following is correct?

A. Is the second most common pattern of disease

B. Represents Type 2 disease

C. Represents Type 3 disease

D. The location determines the category of type (1/2/3)

E. Represents Type 1

A

E. Represents Type 1

Type 2 smaller cysts are 0.5-2cm. Type 3 appears solid but contains multiple small cysts (0.3-0.5cm). Type 1 is the most common.

How well did you know this?
1
Not at all
2
3
4
5
Perfectly
5
Q
  1. Which of the following suggests intralobar rather than extralobar sequestration?’

A. Own pleural covering

B. Systemic venous drainage

C. Symptomatic in first six months

D. Infarction rare

E. Congenital anomalies in 15%

A

E. Congenital anomalies in 15%

Other features of intralobar sequestration includes visceral pleural cavity (pathological findings), and venous drainage to inferior pulmonary vein. Extralobar are associated with 50% of congenital abnormalities.

How well did you know this?
1
Not at all
2
3
4
5
Perfectly
6
Q
  1. Regarding congenital diaphragmatic hernias:

A. Most commonly right-sided

B. Of Bochdalek type are anterior

C. Of Morgagni are posterior

D. Are associated with pulmonary hypoplasia

E. Are mostly diagnosed by postnatal ultrasound

A

D. Are associated with pulmonary hypoplasia

Most congenital diaphragmatic hernias are diagnosed by antenatal ultrasound, and are left-sided. Bochdalek hernias are posterior and common. Morgagni hernias are anterior and rare.

How well did you know this?
1
Not at all
2
3
4
5
Perfectly
7
Q

(Chest) 30. The chest radiograph of a 35-year-old male smoker, performed for immigration purposes, shows hyperlucency of the entire right lung, with a small ipsilateral hilum. Lung markings are seen to the periphery. An expiratory film performed immediately after reveals air trapping. Which is the most likely diagnosis?

A. Hypogenetic lung syndrome

B. MacLeod syndrome

C. Unilateral proximal obstruction of the right pulmonary artery

D. Bronchial obstruction

E. Unilateral bullae

A

B. MacLeod syndrome

MacLeod or Swyer-James syndrome manifests as hyperlucency with features of air trapping. The affected lung in proximal interruption of a pulmonary artery is often as opaque, or slightly more opaque, than the contralateral lung and there is no evidence of air trapping.

How well did you know this?
1
Not at all
2
3
4
5
Perfectly
8
Q

43 Which of the following conditions occurs more frequently in post-maturity rather than pre-term neonates?

(a) Germinal matrix haemorrhage

(b) Meconium aspiration

(c) Necrotizing entero-colitis

(d) Periventricular haemorrhage

(e) Respiratory distress syndrome

A

(b) Meconium aspiration

Meconium is the first stool passed, it is sterile and composed of epithelial cells, mucus, amniotic fluid, bile and water. Aspiration occurs if the meconium is expelled into the amniotic fluid prior to birth, or during labour. It is more common in post-mature births, if there is prolonged labour, and where foetal distress occurs during labour. Postmaturity is also associated with birth trauma and its sequelae, e.g. DDH. The other conditions are associated with prematurity.

How well did you know this?
1
Not at all
2
3
4
5
Perfectly
9
Q

7) A neonate in neonatal intensive care has an abdominal film showing an umbilical arterial catheter (UAC) and an umbilical venous catheter (UVC), both in good position. Which of the following features would help to determine which catheter is which?

a. UAC is smaller in calibre

b. tip of the UAC lies at T8 level

c. UAC courses down into pelvis before approaching thorax

d. UVC is longer than UAC

e. UAC tip always lies to the left of tip of the UVC

A

c. UAC courses down into pelvis before approaching thorax

The UAC passes along the umbilical artery to the internal iliac artery, from which it arises. It then passes up the common iliac artery into the aorta.

The UVC passes up the umbilical vein to the left portal vein, through the ductus venosus, into the middle or left hepatic vein and into the inferior vena cava.

The tips of the catheters may vary in position, but ideal placement of the UVC is around the T8–9 level, with high placement of the UAC being at the T7–9 level.

The tip of the UVC may lie on either side of the UAC depending on whether it reaches the inferior vena cava or remains in the left lobe of the liver.

The UAC and UVC are usually of the same diameter.

Their length is determined by their course, and is not a reliable predictor in identifying whether the catheter is arterial or venous.

How well did you know this?
1
Not at all
2
3
4
5
Perfectly
10
Q

11) A 2-year-old girl presents with respiratory distress following recent treatment for a chest infection that has been slow to resolve. A chest radiograph shows a large, well-defined mass in the lower right thorax. This comprises multiple cysts, with apparent hypoplasia of the ipsilateral lung, and is causing contralateral mediastinal shift. What is the most likely diagnosis?

a. bronchopulmonary sequestration

b. hypogenetic lung syndrome

c. congenital lobar emphysema

d. bronchogenic cyst

e. type I cystic adenomatoid malformation

A

e. type I cystic adenomatoid malformation

Cystic adenomatoid malformation (CAM) is a congenital cystic abnormality of the lung. There are three types: I – single/multiple large cysts of >20 mm; II – multiple cysts of 5–12 mm; III – solitary mass with microcysts. If undetected in utero, they usually present in the first year with respiratory distress and cyanosis, but can present later with recurrent infections. In type I CAM, the chest radiograph shows an expansile mass with multiple air- or fluid-filled cysts, with compression or hypoplasia of ipsilateral lung and contralateral mediastinal shift. Extralobar bronchopulmonary sequestration produces a wedge-shaped mass, usually posteromedially in either lung. Bronchogenic cysts are usually well-defined soft-tissue masses arising from the mediastinum and often present later in life. Hypogenetic lung syndrome produces a small hemithorax with reduced vascularity rather than a mass. Congenital lobar emphysema produces overinflation of one, or occasionally two, lobes, with hyperlucency. This may be mass like at birth due to delayed clearance of fluid.

How well did you know this?
1
Not at all
2
3
4
5
Perfectly
11
Q

@# 14) A 3-day-old boy presents with respiratory distress without cyanosis. Clinically, there is reduced air entry in the right hemithorax with dull percussion note. A chest radiograph shows an opaque right hemithorax with mediastinal shift to the left. Ultrasound scan shows a large effusion, which aspiration demonstrates to be milky. What is the most likely cause?

a. idiopathic

b. birth trauma

c. lymphangioleiomyomatosis

d. thoracic duct atresia

e. lymphangiectasia

A

a. idiopathic

Chylothoraces in neonates are usually right sided, and in most cases no obvious cause is found. Treatment is conservative with special formula and intermittent aspiration. All of the listed conditions are causes of chylothorax, but lymphangioleiomyomatosis presents in adult females and not in the neonatal period.

How well did you know this?
1
Not at all
2
3
4
5
Perfectly
12
Q

16) A newborn is found to have reduced air entry and breath sounds in the right hemithorax, but is otherwise well. A chest radiograph shows an opaque right hemithorax with ipsilateral mediastinal shift. Which feature on ventilation–perfusion scintigraphy would make the diagnosis of pulmonary hypoplasia more likely than complete collapse due to bronchial obstruction?

a. matched marked reduction in ventilation and perfusion

b. more marked reduction in perfusion than ventilation

c. more marked reduction in ventilation than perfusion

d. normal ventilation with reduced perfusion

e. normal perfusion with reduced ventilation

A

a. matched marked reduction in ventilation and perfusion

Pulmonary hypoplasia is the presence of a completely formed but congenitally small bronchus with rudimentary parenchyma and vessels.

This produces a matched marked reduction in ventilation and perfusion or, in severe cases, complete absence of both ventilation and perfusion.

In total lung collapse, the ventilation would be reduced or absent with often reduced, but better, perfusion.

How well did you know this?
1
Not at all
2
3
4
5
Perfectly
13
Q

20) A 7-year-old boy presents with a 1-week history of a cough productive of green sputum. He is pyrexial with a mildly raised white cell count. Which feature on the chest radiograph would be more suggestive of bacterial pneumonia than viral pneumonia?

a. pleural effusion

b. peribronchial cuffing

c. atelectasis

d. airspace opacification

e. cavitation

A

e. cavitation

Bacterial pneumonia is a combination of airway and alveolar disease, whereas viral pneumonia tends to affect airways and peribronchial tissues. Bacterial pneumonia produces two patterns of disease: lobar (consolidation, no volume loss, usually in one lobe, may cavitate) or bronchopneumonic (patchy airspace change which enlarges and coalesces, and volume loss due to mucus plugging). Viral pneumonia tends to produce peribronchial linear densities and an interstitial pattern, though airspace change may be seen in up to 50% of cases. Hilar adenopathy is not often seen, but effusions are present in 20% of cases. Pneumatoceles, pneumothorax and cavitation do not occur. Viral pneumonia may sometimes be complicated by bacterial pneumonia.

How well did you know this?
1
Not at all
2
3
4
5
Perfectly
14
Q

(Chest) 26) A 5-year-old female presents with intermittent, colicky, epigastric discomfort with no specific features. A CT shows transverse colon protruding through a small defect anteriorly in the diaphragm in a parasternal position. What is the most likely diagnosis?

a. Bochdalek’s hernia

b. Morgagni’s hernia

c. rolling hiatus hernia

d. eventration

e. septum transversum defect

A

b. Morgagni’s hernia

A Morgagni hernia is due to a developmental defect anteromedially in the diaphragm between the septum transversum and left (10%) or right (90%) costal margins. They are usually asymptomatic but can produce pain. They are usually small and present in older children or adults.

A Bochdalek hernia is a posterolateral developmental defect. It tends to be large, is more common on the left and presents in infancy.

Rolling hiatus hernia is herniation of the stomach through the oesophageal hiatus alongside the esophagus.

A septum transversum defect is a defect in the central tendon.

How well did you know this?
1
Not at all
2
3
4
5
Perfectly
15
Q

29) In hyaline membrane disease, which is the first feature usually seen on a chest radiograph in the initial stages?

a. reduced lung volumes

b. bilateral consolidation

c. granularity in both lungs

d. pleural effusions

e. white-out of lungs

A

c. granularity in both lungs

Hyaline membrane disease is due to deficiency of pulmonary surfactant, which causes alveolar collapse.

Prematurity, caesarean section and perinatal asphyxia are predisposing factors.

In the mild form, granularity is seen throughout the lungs as the first sign.

As the condition progresses, air bronchograms appear with eventual complete opacification of the lungs.

Changes are usually symmetrical if the condition is uncomplicated.

How well did you know this?
1
Not at all
2
3
4
5
Perfectly
16
Q

(Chest) 41) A 40-year-old female with known history of seizures and low IQ presents with severe shortness of breath, progressively worsening over a number of years. A chest radiograph shows extensive honeycombing throughout both lungs, which are of normal volume. A small pneumothorax is also seen on the right side. What is the most likely diagnosis?

a. sarcoidosis

b. cystic fibrosis

c. tuberous sclerosis

d. lymphangiomyomatosis

e. idiopathic pulmonary fibrosis

A

c. tuberous sclerosis

The lung is involved in 5% of patients with tuberous sclerosis, with symptoms usually occurring in adult life, and it is predominantly women who are affected. There is interstitial fibrosis in the lower lungs with miliary nodular changes, which progress to honeycomb lung.

Recurrent pneumothoraces occur in 50% of cases.

Chylothorax may also be seen. Preservation of the lung volumes with honeycombing is seen in histiocytosis, neurofibromatosis and lymphangiomyomatosis, as well as tuberous sclerosis.

How well did you know this?
1
Not at all
2
3
4
5
Perfectly
17
Q

44) An 18-month-old girl has a cough. A chest radiograph shows consolidation in the right upper lobe. This is treated and a follow-up chest radiograph is performed, which shows a triangular-shaped mass arising from the mediastinum projecting over the right upper lobe. This has a rippled border. What is the most appropriate next investigation?

a. CTof chest

b. blood film

c. MR of chest

d. denatured red blood cell scintigraphy

e. no further investigation

A

e. no further investigation

The appearance is of a normal thymus, which is visible in 50% of children up to 2 years of age. It is usually seen as a triangular density arising from the superior border of the mediastinum (‘sail sign’), which has a rippled border due to indentation by the ribs (‘wave sign’). The shape may change with position and respiration.

How well did you know this?
1
Not at all
2
3
4
5
Perfectly
18
Q

(CVS) 53) A 25-year-old male presents with dyspnea on exertion, cough and hemoptysis. He has a history of recurrent chest infections as a child. A chest radiograph shows a hyperlucent left lung. A pulmonary embolus is suspected and a _V= _Q scan is arranged. This shows reduced perfusion and ventilation of the left lung, with delayed washout on ventilation. What is the most likely diagnosis?

a. acute pulmonary embolus

b. Macleod’s syndrome

c. congenital lobar emphysema

d. Poland’s syndrome

e. hypogenetic lung syndrome

A

b. Macleod’s syndrome

Macleod’s (Swyer–James) syndrome is a result of acute bronchiolitis in childhood causing obliterative bronchiolitis. There is usually a history of recurrent infections. Chest radiographs show hyperlucent lung with a small or normal-sized hemithorax. High-resolution CT shows reduced vascularity and attenuation of lung with air trapping.

Congenital lobar emphysema presents in the first 6 months of life with respiratory distress and a hyperlucent, overexpanded lobe (usually left upper) on the chest radiograph.

Poland’s syndrome is an absence of part of the pectoralis major muscle, which causes apparent hyperlucent lung, though the lung is normal.

Hypogenetic lung syndrome is usually asymptomatic and causes reduced volume of lung with reduced lucency.

How well did you know this?
1
Not at all
2
3
4
5
Perfectly
19
Q

56) A neonate presents with respiratory distress shortly after birth. A chest radiograph shows hazy opacification in the left upper zone, which improves over the next 7 days, and the chest radiograph becomes normal. Two months later, the baby re-presents with respiratory distress, and a chest radiograph shows hyperlucency in the left upper zone with contralateral mediastinal shift and compression of the adjacent left lower lobe. What is the likely diagnosis?

a. congenital lobar emphysema

b. Macleod’s syndrome

c. bronchiolitis obliterans

d. foreign body

e. carcinoid

A

a. congenital lobar emphysema

Congenital lobar emphysema results in progressive overinflation of one or more pulmonary lobes; it presents in the first 6 months of life with respiratory distress and cyanosis. The left upper lobe is most commonly affected. Imaging features immediately after birth are of a hazy, mass-like opacity, which represents delayed clearing of lung fluid in the emphysematous lobe. After clearing of the fluid, the affected lobe becomes expanded and hyperlucent on imaging, and causes mass effect and mediastinal shift.

Macleod’s syndrome is a complication of bronchiolitis, usually presenting with hyperlucency of one or both lungs.

Bronchiolitis obliterans usually occurs in adults, but may present in children following infection.

How well did you know this?
1
Not at all
2
3
4
5
Perfectly
20
Q

59) A 3-year-old boy presents with a cough of 1 week’s duration. There is no relevant preceding history. A chest radiograph shows lucency in the right lower zone, and an expiratory film demonstrates air trapping in the same region as well as mediastinal shift to the left. No masses are seen. What is the likely diagnosis?

a. foreign body aspiration

b. congenital lobar emphysema

c. congenital cystic adenomatoid malformation type I

d. bronchiolitis obliterans

e. bronchogenic cyst

A

a. foreign body aspiration

Foreign body aspiration is most common in children under 3 years of age. It presents with varying degrees of cough, and the chest radiograph usually shows overinflation, atelectasis, infiltrates and/or air trapping, which almost exclusively involve the lower lobes, with the right most commonly affected. A foreign body is seen in only about 9% of cases.

Congenital lobar emphysema presents with increased lucency, usually in the upper zones, with air trapping and mediastinal shift.

Cystic adenomatoid malformation type I usually presents as an expansile mass with multiple cysts.

Bronchiolitis obliterans tends to present in adults, though it may present in children, usually with generalized hyperinflation and patchy air trapping.

How well did you know this?
1
Not at all
2
3
4
5
Perfectly
21
Q

64) A 10 month old presents with recurrent chest infections, the most recent caused by Pseudomonas aeruginosa. The chest radiograph shows patchy infiltrates in the right base. Which feature on high resolution CT would make the diagnosis of cystic fibrosis more likely than Williams–Campbell syndrome?

a. cystic bronchiectasis

b. emphysematous bullae

c. mucus plugging

d. lower lobe involvement

e. pleural effusions

A

c. mucus plugging

Cystic fibrosis usually presents in the first year of life with cough and recurrent infections or progressive respiratory insufficiency. The upper lobes are predominantly affected. High-resolution CT shows mucus plugging (tubular opacities), bronchiectasis, peribronchial thickening, bullous formation and collapse/consolidation.

Williams–Campbell syndrome is a congenital deficiency of cartilage in the fourth to sixth generation bronchi. This produces cystic bronchiectatic changes beyond 3rd bronchial generation and bullous changes. Mucus plugging is not a feature.

How well did you know this?
1
Not at all
2
3
4
5
Perfectly
22
Q

74) A neonate is delivered following an uncomplicated pregnancy and presents with respiratory distress but no cyanosis. No resuscitation or ventilation is required. A chest radiograph shows a pneumothorax, which is treated by aspiration. What investigation should be considered?

a. cranial ultrasound scan

b. renal ultrasound scan

c. abdominal radiograph

d. barium swallow

e. ascending urethrogram

A

b. renal ultrasound scan

Spontaneous pneumothorax may occur in babies where there are renal anomalies, and routine ultrasound scan is recommended. This is often associated with maternal oligohydramnios, but this may not necessarily be present.

How well did you know this?
1
Not at all
2
3
4
5
Perfectly
23
Q

77) A 30-week premature baby is delivered normally and shortly after birth develops tachypnoea and expiratory grunting. What feature on the chest radiograph would make respiratory distress syndrome of the newborn more likely than meconium aspiration syndrome?

a. bilateral consolidation

b. pneumothorax

c. pleural effusion

d. air bronchograms

e. hyperinflation with air trapping

A

d. air bronchograms

Respiratory distress syndrome of the newborn is the commonest cause of respiratory distress in premature neonates, and is due to relative immaturity of type II pneumocytes. Features seen on chest radiograph are reduced lung expansion, bilateral and symmetrical consolidation, and prominent air bronchograms. These resolve over several days.

Meconium aspiration syndrome is the commonest cause of respiratory distress in full-term neonates. Hyperinflation, pneumothorax and pleural effusions are seen, as are diffuse patchy opacities due to atelectasis and consolidation. No air bronchograms are seen. Changes usually resolve in 48 hours.

How well did you know this?
1
Not at all
2
3
4
5
Perfectly
24
Q

4 A 1 year old boy presents with an acute history of cough, wheeze, and respiratory distress. Which of the following is true?

(a) Collapse of a lung or lobe is the commonest radiographic finding acutely

(b) An inhaled foreign body is equally likely to pass into either mainstem bronchus

(c) The majority of foreign bodies are radio-opaque and can be seen on CXR

(d) Mediastinal shift on expiratory radiographs/fluoroscopy is toward the lucent side

(e) Normal imaging excludes the diagnosis of inhaled foreign Body

A

(b) An inhaled foreign body is equally likely to pass into either mainstem bronchus

The majority of inhaled foreign bodies are food material (e.g. peanuts), thus are not radio-opaque.

The paediatric airway diameter increases inspiration, decreases in expiration - a partial obstruction in inspiration may become complete in expiration, allowing air-trapping in expiration.

The carinal angle is symmetric in infants making it equally likely that a foreign body will pass into either left or right mainstem bronchus.

Radiographic evidence of air trapping is typical, however, no imaging modality can definitely exclude the diagnosis, therefore, if the history is suggestive, the child should undergo bronchoscopy.

How well did you know this?
1
Not at all
2
3
4
5
Perfectly
25
Q

12 An 8 month old boy presents with prolonged cough. On examination there is reduced air entry on the right side. The subsequent CXR demonstrates mediastinal shift towards the left side of the chest. Which of the following is more likely?

(a) Congenital lobar emphysema

(b) Bronchogenic cyst

(c) Pulmonary agenesis

(d) Scimitar syndrome

(e) Swyer-James (Mcleod) syndrome

A

(a) Congenital lobar emphysema

Causes of mediastinal shift away from the abnormal side include pleural effusions, tumours, diaphragmatic hernia, tension pneumothorax, and causes of air-trapping (e.g. bronchial atresia, congenital lobar emphysema). Asthma may be a cause, but uncomplicated cases do not cause shift. Mediastinal shift towards the abnormal hemithorax suggests volume loss as a cause (e.g. lobar collapse).

How well did you know this?
1
Not at all
2
3
4
5
Perfectly
26
Q

(Chest) 23- An 18-year-old man presents with a long history of recurrent pneumonias- CXR shows a 6 cm right lower lobe cystic lesion with an air/ fluid level within it. Contrast enhanced CT shows a single lesion containing fluid of water density along with higher attenuation material within it. A single aberrant vessel arising from the distal thoracic aorta is seen to enter the lesion. Angiography confirms this vessel and also demonstrates venous drainage via the pulmonary veins into the left atrium What is the likeliest diagnosis?

(a) Intra lobar sequestration

(b) Extra lobar sequestration

(c) Lateral thoracic meningocele

(d) Extramedullary hematopoiesis

(e) Pulmonary abscess

A

(a) Intra lobar sequestration

Intralobar sequestration is congenital malformation involving a lesion enclosed within the visceral pleura of the affected pulmonary lobe but has no connection with the tracheobronchial tree. It is more common than extralobar sequestration and can present in adulthood. Venous drainage of the lesion is via the pulmonary as opposed to the systemic veins as seen in extralobar sequestration. CT shows single or multiple cystic lesions filled with a combination of fluid, pus and mucus.

How well did you know this?
1
Not at all
2
3
4
5
Perfectly
27
Q

30 A 3 day old baby has dyspnoea. The CXR shows shift of the mediastinum to the right; the left hemithorax is filled with multiple cyst-like structures. The stomach is centrally located and there is a paucity of bowel gas within the abdomen. What is the most likely diagnosis?

(a) Bochdalek hernia

(b) Bronchogenic cyst

(c) Congenital cystic adenomatoid malformation

(d) Extrapulrlnbnary sequestration

(e) Septum transverum defect

A

(a) Bochdalek hernia

Congenital diaphragmatic hernias include Bochdalek (85-90%), Morgagni (5%), and rarely: septum transverum defects, hiatus hernia and diaphragmatic eventration.

Bochdalek hernias are posterior, and left-sided (80%),

Morgagni hernias are anterio-medial, right-sided.

Differentiation from CCAM can be difficult, however, CCAM is less commonly macrocystic, the stomach will be normally sited to the left and the bowel gas pattern will be normal.

How well did you know this?
1
Not at all
2
3
4
5
Perfectly
28
Q

32 A 3 week old, previously well baby present with cough and difficulty in breathing. On examination there is reduced air entry on the right. CXR reveals complete opacification of the right hemithorax. There is rib crowding on the right and the trachea is positioned to the right. What is the most likely underlying cause?

(a) Congenital cystic adenomatoid malformation

(b) Congenital diaphragmatic hernia

(c) Extra-pulmonary sequestration

(d) Mucous plugging

(e) Pleural effusion

A

(d) Mucous plugging

When there is a unilateral dense hemithorax, it needs to be decided if this is the abnormal side (usually obvious). The differential diagnosis is based on the medisatinum: if it moves towards the opaque side it suggests volume loss (collapse due to mucous plugging, asthma, CF, ET tube misplacement, pulmonary aplasia, and, rarely, pneumonectomy). Mediastinal shift away from the opaque side suggests mass effect, usually due to pleural fluid (or tumours, CCAM, etc).

How well did you know this?
1
Not at all
2
3
4
5
Perfectly
29
Q

33 Regarding lung lesions in the paediatric population. Which of the following conditions is the most likely to present as multiple pulmonary opacities?

(a) Arterio-venous malformations

(b) Congenital cystic adenomatoid malformation

(c) Extra lobar pulmonary sequestrations

(d) lntralobar pulmonary sequestrations

(e) Pulmonary hamartomas

A

(a) Arterio-venous malformations

AVMs are multiple in up to 35%, and are more likely to be so if associated with hereditary haemorrhagic telangiectasia. Pulmonary sequestrations and pulmonary hamartomas are considered solitary intrathoracic masses.

How well did you know this?
1
Not at all
2
3
4
5
Perfectly
30
Q

36 A 1 year old girl presents to A&E with a harsh cough, fever and stridor. A lateral neck X-ray shows normal supraglottic structures. The CXR shows narrowing of the sub-glottic airway. What is the most likely diagnosis?

(a) Croup

(b) Enlarged tonsils

(c) Epiglottitis

(d) Inhaled foreign body in RLL

(e) Vocal cord palsy

A

(a) Croup

Croup is a viral infection occurring at 6 months - 3 years, mainly in winter. The ‘steeple’ (or inverted ‘V’) sign is due to oedematous narrowing of the sub-glottic airway.

Epiglottitis is caused by , Haemophilus influenzae and typically presents with stridor, fever, sore throat and drooling. It usually presents in older children (3-6 years), the key finding is thickened aryepiglottic folds on lateral neck films and it may be associated with subglottic narrowing similar to croup.

How well did you know this?
1
Not at all
2
3
4
5
Perfectly
31
Q

1 In a neonate with respiratory distress, which of the following CXR findings are more indicative of a diagnosis of meconium aspiration over hyaline membrane disease?

(a) Air bronchograms

(b) Ground glass changes

(c) Humeral heads not ossified

(d) Hyperinflation

(e) No pleural fluid

A

(d) Hyperinflation

In HMO the lungs are hypoinflated due to surfactant deficiency and hypoaeration (unless the patient is intubated). In meconium aspiration the lungs may be of normal volume, but are typically hyperinflated due to air trapping. The other findings are non-specific and may be present in both conditions.

How well did you know this?
1
Not at all
2
3
4
5
Perfectly
32
Q

4 Regarding congenital lobar emphysema. Which lobe is most commonly affected?

(a) Left lower lobe

(b) Left upper lobe

(c) Right middle lobe

(d) Right lower lobe

(e) Right upper lobe

A

(b) Left upper lobe

The most commotiily affected lobes, in order of decreasing frequency are: LUL (42-42%), RML (32-35%), RUL (20%). Two or more lobes are only affected in 5% of cases.

How well did you know this?
1
Not at all
2
3
4
5
Perfectly
33
Q

5 A 2 year old child presents with an acute history of coughing and wheezing. On examination, there is reduced air entry on the right side. Which single CXR view would you advise to maximise the chances of confirming the suspected diagnosis?

(a) Expiratory film

(b) Left lateral decubitus film

(c) PA erect film

(d) Prone film

(e) Supine film

A

(a) Expiratory film

The triad of cough, wheeze and reduced air entry in a child of appropriate age (6 months-4 years) is classic for an inhaled foreign body. Most are radiolucent, paired inspiratory/ expiratory CXRs are the most sensitive, however, the child may not cooperate. Lateral decubitus views may show air trapping, but the abnormal lung should be dependent. Fluoroscopy can show air-trapping dynamically, CT can also be considered prior to bronchoscopy.

How well did you know this?
1
Not at all
2
3
4
5
Perfectly
34
Q

15 Which of the following is most commonly associated with cystic fibrosis?

(a) Low sodium concentration in sweat

(b) Meconium plug syndrome

(c) Pectus excavatum

(d) Pneumothorax

(e) Situs inversus

A

(d) Pneumothorax

Pneumothorax is common and may be recurrent, due to rupture of bulla/blebs. Kartagener’s syndrome is associated with situs inversus. Sweat has high sodium and chloride levels. Meconium ileus, not meconium plug syndrome, is associated. Like asthma, CF is associated with pectus carinatum.

How well did you know this?
1
Not at all
2
3
4
5
Perfectly
35
Q

13 A 2 year old girl presents with fatigue, loss of appetite and fever. A CXR done as part of a septic screen shows a centrally placed mass with loss of the right paratracheal stripe, and subtle splaying of the ribs. The hilar structures and right heart border are clearly seen. What is the most likely diagnosis?

(a) Germ cell tumour

(b) Hodgkin’s Disease

(c) Leukaemia

(d) Neuroblastoma

(e) Non-Hodgkin’s Lymphoma

A

(d) Neuroblastoma

Pyrexia of unknown origin has a malignant aetiology in 10% of cases (e.g. leukaemia, lymphoma, neuroblastoma, hepatoma, or sarcoma in children). A posterior mediastinal mass is described, lymphoma/ leukaemia can present like this, but they are classically in the anterior mediastinum; in this case the most likely diagnosis is neuroblastoma.

How well did you know this?
1
Not at all
2
3
4
5
Perfectly
36
Q

18 A young child presents in great distress with breathing difficulties. Which of the following would be more typical of croup rather than epiglottitis?

(a) The child is 1 year old at presentation

(b) Predominantly supraglottic involvement

(c) The child is febrile

(d) Associated dysphagia

(e) Increased stridor when placed supine

A

(a) The child is 1 year old at presentation

Croup is characterised by oedema of the glottis and subglottis, producing a ‘steeple shaped’ laryngeal airway on AP radiograph. It is not typically associated with either fever or dysphagia. Epiglottitis occurs in older children.

How well did you know this?
1
Not at all
2
3
4
5
Perfectly
37
Q

32 A 3 year old presents with a persistent cough that has not responded to antibiotics. CXR demonstrates a triangular shaped-mass medially at the left lung base. CT is arranged for further investigation. What feature favours a diagnosis of intra- over extrapulmonary sequestration?

(a) Air seen within lesion

(b) Communication with the bronchial tree

(c) Multiple associated anomalies

(d) Systemic venous drainage

(e) Systemic arterial supply

A

(a) Air seen within lesion

Intralobar type is within the lung, sharing its pleura; it may be of airless, or air-containing, cystic type, and presents late in childhood / adulthood with recurrent infections or haemoptysis. Arterial supply is systemic, drainage is to the pulmonary vein in 95% and there is a low association with other anomalies. Extrapulmonary sequestration presents in neonates, is located outside the lung with its own pleura, has a systemic arterial supply, and drainage is to the systemic circulation. In 65% there are associated anomalies and it is ‘always’ airless (unless there is a communication with the GI tract).

How well did you know this?
1
Not at all
2
3
4
5
Perfectly
38
Q

37 Which of the following features is not a typical finding of the normal infant thymus on CXR?

(a) Wavy inferior margin

(b) Change in size on inspiratory/ expiratory films

(c) No mass effect

(d) Convex lateral borders

(e) Homogeneous density

A

(a) Wavy inferior margin

The normal thymus widens on expiration, and narrows/ elongates on inspiration. <5 yrs there are smooth biconvex lateral borders, with increasing age the lateral margins straighten or become concave. The anterio-lateral margins can be indented by anterior rib ends to give a scalloped ‘wavy’ appearance laterally, but not inferiorly. The right lobe often has a flattened inferior border near the horizontal fissure producing the ‘sail sign’.

How well did you know this?
1
Not at all
2
3
4
5
Perfectly
39
Q

45 CXR shows asymmetrical density, with the right hemithorax more translucent that the left and there is noted to be normal and symmetrical pulmonary vascularity. lnspiratory/ expiratory films show symmetrical size changes of the hemithoraces. What is the most likely diagnosis?

(a) Congenital lobar emphysema

(b) Large pneumatocoele

(c) Pneumothorax

(d) Poland’s syndrome

(e) Pulmonary artery hypoplasia

A

(d) Poland’s syndrome

If there is asymmetric transradiancy and the lungs are abnormal, there are 3 factors to consider: differences in aeration, vascularity and size. If there is different vascularity, the side with the decreased vascularity is abnormal. If there are differences in aeration, the side which changes least on expiration is usually abnormal. In this case the lungs are normal and the increased translucency must be due to another cause. Poland’s syndrome has underdevelopment or absence of the pectoral muscle unilaterally (typically right); there may be associated webbing of the fingers.

How well did you know this?
1
Not at all
2
3
4
5
Perfectly
40
Q

(Chest) 51 A 35 year old lady undergoes a screening CXR. This shows unilateral transradiancy with reduced vessel markings on the affected side. A subsequent CT of the thorax reveals the changes are bilateral but asymmetrical, with areas of bronchiectasis. What is the most likely diagnosis?

(a) Bronchial atresia

(b) Congenital lobar emphysema

(c) Panacinar emphysema

(d) Swyer-James (Mcleod) syndrome

(e) Primary ciliary dyskinesia

A

(d) Swyer-James (Mcleod) syndrome

These are the typical features of Swyer-James (McLeod’s) syndrome, a form of obliterative bronchiolitis that follows an insult (most commonly viral) to the developing lung. The CT findings are often more extensive than the plain film, with bilateral changes common.

How well did you know this?
1
Not at all
2
3
4
5
Perfectly
41
Q

55 A 2 year old child is seen by her GP following recurrent chest infections. A CXR is performed and the thymus can be seen to extend down to the diaphragm. Which of the following is the least likely cause?

(a) Cushing’s disease

(b) Recovery from illness

(c) DiGeorge syndrome

(d) Hyperthyroidism

(e) Normal

A

(c) DiGeorge syndrome

Rebound hyperplasia is seen during recovery from illness, after chemotherapy or radiotherapy or following treatment for Cushing’s disease.

Hypoplasia or aplasia of the thymus is seen in DiGeorge syndrome.

In infancy a prominent thymus is almost always normal and can occasionally extend as far as the diaphragm.

How well did you know this?
1
Not at all
2
3
4
5
Perfectly
42
Q

61 A 6 hour old term baby delivered by Caesarian section has increased respiratory rate with intercostal recession. CXR shows prominent vascular markings, Kerley B lines and small bilateral pleural effusions. There is full resolution of symptoms and CXR findings after 2 days. What is the most likely diagnosis?

(a) Hyaline membrane disease

(b) Meconium aspiration

(c) Neonatal pneumonia

(d) Transient tachypnoea of the newborn

(e) Tricuspid atresia

A

(d) Transient tachypnoea of the newborn

TTN is the commonest cause of respiratory distress in the newborn. Risk factors include C-section, breech, or fast deliveries (thus fluid is not ‘squeezed’ from the lungs through the birth canal). Onset is typically within the first 6 hours (peak 1 day). The CXR findings given are typical and although there can be overlap, the history/ risk factors and subsequent resolution make TTN the most likely diagnosis.

How well did you know this?
1
Not at all
2
3
4
5
Perfectly
43
Q

68 A 14 day old neonate born at 32 weeks has been treated for respiratory distress syndrome and has remained on a ventilator. Increasing ventilatory pressures have been required. The CXR shows overinflated lungs, black ‘streaks’ running in the line of the bronchi but a normal heart size. What is the most likely diagnosis?

(a) Broncho-pulmonary dysplasia

(b) Patent ductus arteriosus

(c) Pulmonary haemorrhage

(d) Pulmonary interstitial emphysema

(e) Wilson Mikity Syndrome

A

(d) Pulmonary interstitial emphysema

PIE is typically due to positive-pressure ventilation leading to air leaks from the bronchi to the interstitium. PIE makes the lungs ‘stiff’ and more difficult to ventilate and leads to overinflated lungs, with the potential for pneumothorax or pneumo-mediastinum. BPD is diagnosed clinically as oxygen dependency at 28 days of age or 36 corrected weeks of gestation.

How well did you know this?
1
Not at all
2
3
4
5
Perfectly
44
Q
  1. A four year old girl presents with persistent left upper lobe pneumonia with a fingerlike opacity projecting from the hilum. The most likely diagnosis is:

a. Bronchial atresia

b. Intralobar sequestration

c. Staphylococcal pneumonia

d. Congenital lobar emphysema

e. Bronchogenic cyst

A
  1. a. Bronchial atresia

The site and features described are characteristic of bronchial atresia.

How well did you know this?
1
Not at all
2
3
4
5
Perfectly
45
Q
  1. A five month old baby presents with failure to thrive and respiratory distress. Plain radiograph demonstrates a left basal homogenous opacity devoid of air bronchograms. Which of the following is least likely on further imaging?

a. Radionuclide angiography does not demonstrate perfusion in the pulmonary phase

b. Invested in its own pleura

c. Co-existing anomalies are common

d. It is usually supplied by branches from the descending aorta

e. Commonly drains into the left atrium

A
  1. e. Commonly drains into the left atrium

Extralobar sequestration is usually seen in early childhood and demonstrates all the above features, but the venous drainage is into the right atrium through systemic veins.

How well did you know this?
1
Not at all
2
3
4
5
Perfectly
46
Q

(Chest) 14. A 26 year old undergoes a routine chest radiograph as part of the Australian residency application. The left upper lobe is hyperlucent and hyperexpanded and a lobular mass is demonstrated adjacent to the left hilum. CT reveals the presence of a dilated bronchus containing a plug of soft tissue. The surrounding lung is emphysematous. The most likely diagnosis is:

a. Central carcinoid tumour

b. Bronchogenic cyst

c. Bronchial atresia

d. Cystic adenomatoid malformation

e. Congenital lobar emphysema

A
  1. c. Bronchial atresia

The case describes bronchial atresia with mucoid impaction. Bronchial atresia is a congenital abnormality that is usually discovered incidentally. It results in local obliteration of the proximal lumen of a segmental bronchus. The apicoposterior segment of the left upper lobe is most commonly affected. Airways distal to the atretic segment continue to produce mucous which can lead to mucoid impaction/mucocoele. The airways distal to the atretic segment also develop normally and are ventilated by collateral air shift. This results in the affected lobe appearing hyperexpanded, oligaemic and hyperlucent.

Congenital lobar emphysema can look similar, however there is usually no mucous plug and patients tend to present early.

How well did you know this?
1
Not at all
2
3
4
5
Perfectly
47
Q
  1. A 14 year old with thalassaemia presents with mild breathlessness. The only abnormality on the chest radiograph is a well-rounded opacity without any air bronchograms. The likely location would be:

a. Perihilar

b. Anterior mediastinal

c. Abutting the chest wall

d. Paravertebral

e. Apical

A
  1. d. Paravertebral

Extramedullary haemopoiesis can present as uni/bilateral rounded masses in the lower paravertebral region commonly between T8 and T12.

How well did you know this?
1
Not at all
2
3
4
5
Perfectly
48
Q
  1. A four year old child with a known malignancy presents with multiple pulmonary metastases. Which of the following is the most likely radiological description of the primary lesion?

a. CT of the abdomen demonstrating a large mass in the left flank displacing the kidney inferiorly with stippled calcification

b. CT of the abdomen demonstrating a large low-attenuation hepatic mass pre-contrast which demonstrates early and avid enhancement post-contrast

c. CT of the abdomen demonstrating a low-attenuation hepatic mass with rim enhancement

d. Plain radiograph of the right femur revealing a moth-eaten permeative lesion

e. A well-circumscribed heterogeneous mass in the left kidney which enhances to a lesser degree than the kidney

A
  1. e. A well-circumscribed heterogeneous mass in the left kidney which enhances to a lesser degree than the kidney

Neuroblastoma (a) presents earlier and is more likely to metastasise to the liver, whilst this is the right age for Wilms tumour (e) to metastasise to the lung.

Haemangioendothelioma (b) presents in early infancy with heart failure.

Hepatoblastoma (c) also presents earlier and is less common than Wilms tumour, although it does metastasise to the lung.

How well did you know this?
1
Not at all
2
3
4
5
Perfectly
49
Q
  1. A neonate presents with respiratory distress a week after birth. Plain radiograph demonstrates a hazy opacity in the left upper zone. Follow-up radiograph a few days later demonstrates clearing up of the opacity noted previously and a hyperlucent underlying lung with evidence of a contralateral shift. Which of the following is not a likely cause for the findings?

a. Bronchial dysplasia

b. Inspissated mucous

c. Patent ductus arteriosus

d. Anomalous origin of the right subclavian artery

e. Bronchial web

A
  1. d. Anomalous origin of the right subclavian artery.

The features described are those of congenital lobar emphysema, which may result from all the above apart from anomalous origin of the right subclavian artery, which usually does not result in tracheobronchial indentation.

How well did you know this?
1
Not at all
2
3
4
5
Perfectly
50
Q
  1. A 15 year old girl referred for a chest radiograph demonstrates a large well-rounded mediastinal mass. CT suggests a teratoma. Which of the following is true?

a. They are often inseparable from the thymus

b. They are always anterior mediastinal

c. Rim enhancement indicates malignancy

d. A fat-fluid level is a common and specific sign

e. Homogenous soft-tissue density precludes a diagnosis of teratoma

A
  1. a. They are often inseparable from the thymus

Mature teratoma can be seen in the posterior mediastinum and may demonstrate rim enhancement. A fat-fluid level is a specific sign but is uncommon. They may occasionally have a homogenous soft-tissue appearance and be indistinguishable from lymphoma.

How well did you know this?
1
Not at all
2
3
4
5
Perfectly
51
Q
  1. A seven year old girl with repeated chest infections and chronic cough presents with another episode of acute exacerbation. She is known to have raised sodium and chloride in her sweat. Which of the following features is least likely on an HRCT of her chest?

a. Cylindrical bronchiectasis

b. Centrilobular emphysema

c. Segmental/subsegmental atelectasis

d. Branching intrabronchial soft tissue

e. Hilar lymphadenopathy

A
  1. b. Centrilobular emphysema

All the features described are of cystic fibrosis except centrilobular emphysema. They usually develop paraseptal emphysema.

How well did you know this?
1
Not at all
2
3
4
5
Perfectly
52
Q

(Chest) 50. A 30 year old female patient with a history of recurrent lower respiratory tract infections as a child presents with cough and dyspnoea. Chest radiograph demonstrates a smaller hyperlucent left lung. Which of the following features is unlikely to be seen on HRCT?

a. Air trapping

b. Small left hemithorax

c. Diminished size of pulmonary vessels

d. Bronchiectasis

e. Left hilar enlargement

A
  1. e. Left hilar enlargement

The case describes Swyer–James syndrome (Macleod syndrome). This is a post-infectious constrictive bronchiolitis which causes a small, hyperlucent lung with bronchiectasis and air trapping in expiration. The number and size of pulmonary vessels are also diminished,resulting in a small ipsilateral hilum.

53
Q

QUESTION 3
A 3-day-old neonate becomes dyspnoeic and requires a chest radiograph. Which one of the following statements will be true regarding the chest radiograph in this neonate?

A The ribs are more vertical than those in an older child’s radiograph.

B A normal thymus may compress mediastinal structures.

C The cardiothoracic ratio (CTR) may be as large as 80%.

D The thymus may involute if the child is given steroid treatment.

E The sternum is fully ossified.

A

D The thymus may involute if the child is given steroid treatment.

The neonatal chest radiograph differs from the infant; the ribs tend to be more horizontal, resulting in a cylindrical shape and the CTR can be as large as 65%.

54
Q

QUESTION 9
A newborn delivered by caesarean section shows signs of respiratoiy distress soon after birth. A chest radiograph is performed. Which one of the following features favours the diagnosis of transient tachypnoea of the newborn (TTN)?

A A ground glass appearance throughout both lungs

B Hyper inflated lungs

C Loss of lung volume

D Radiographic resolution after 2 weeks

E The presence of a pleural effusion

A

B Hyper inflated lungs

The radiographic features of TTN include hyperaeration of the lungs with an increase in pulmonary interstitial markings. Resolution should be seen clinically and radiographically within 48—72 hours.

55
Q

QUESTION 12
A 4-year-old child presents with upper back pain. General examination reveals hepatomegaly and blood tests demonstrate an iron deficiency anaemia. The child’s chest radiograph demonstrates an abnormal mediastinal contour and subsequent CT confirms an 8-cm posterior mediastinal mass which contains calcification. The lungs are clear. Which one of the following is most likely the diagnosis?

A Extramedullary haemopoiesis

B Lymphoma

C Neuroblastoma

D Neurofibroma

E Teratoma

A

C Neuroblastoma

If a paediatric posterior mediastinal mass contains calcification, it is most likely to be a sympathetic chain tumour.

56
Q

(Chest) QUESTION 17
A 25-year-old man has a CXR (PA and lateral) performed for a chronic cough. This demonstrates a mass projected anterior to the ascending aorta and a contrast-enhanced CT chest is performed. There are no associated clinical syndromes (such as myasthenia gravis) and no CT features to suggest a thymic mass or germ cell tumour. What additional CT finding is most likely to suggest a diagnosis of lymphangioma?

A Bone destruction

B Multiple cystic spaces

C Narrow contact with the ascending aorta

D Retrosternal extension

E Uniform fat attenuation

A

B Multiple cystic spaces

Lymphangiomas (cystic hygromas) are congenital malformations of the lymphatic system presenting as prevascular masses and comprising complex cystic spaces with the attenuation of the contents close to water on CT.

57
Q

QUESTION 24
A 3-day-old neonate demonstrates signs of respiratory distress. A chest radiograph demonstrates a right pleural effusion. Which of the following is the commonest cause?

A Chylothorax

B Hydrops fetalis

C Meconium aspiration syndrome

D Pulmonary haemorrhage

E Respiratory distress syndrome

A

A Chylothorax

Pleural effusions are unusual in the newborn; other causes include hydrops fetalis, cardiac failure and perinatal infection.

58
Q

QUESTION 29
Immediately following delivery, a neonate experiences respiratory difficulties. On examination the trachea is deviated to the left, and there is reduced air entry on the right. A chest radiograph confirms mediastinal deviation to the left with an opaque right hemithorax. A NG tube is passed and a further chest radiograph is performed. This demonstrates that the stomach lies within the chest cavity. Which one of the following best describes the most common site and size of a congenital diaphragmatic hernia?

A Anterior, small and left sided

B Anterior, small and right sided

C Posterior, large and left sided

D Posterior, large and right sided

E Posterior, small and right sided

A

C Posterior, large and left sided

Congenital hernias are usually Bochdalek hernias. Morgagni hernias are usually right-sided, anterior and smaller and tend to occur later on in life.

59
Q

QUESTION 43
It has been decided that a newborn baby requiring respiratory support also requires increased monitoring via arterial and venous umbilical catheters. Radiographs have been taken to confirm their positions. Which of the following statements is true?

A The tip of the arterial line should lie above T6.

B The tip of the arterial line should lie between T10 and L3.

C The tip of the venous line should lie within the liver.

D The tip of the venous line should lie within the superior vena cava (SVC).

E The venous line should pass through the left portal vein.

A

E The venous line should pass through the left portal vein.

The umbilical arterial line should lie between T6 and T10 or between L3 and L5, in order to avoid the origins of the spinal and renal arteries.

60
Q

QUESTION 52
A newborn baby is hypoxic immediately following delivery. There is evidence of meconium-stained amniotic fluid. Which one of the following statements is true regarding meconium aspiration syndrome?

A Radiological resolution is usually seen within 48—72 hours.

B Pneumothorax and pneumomediastinum are uncommon complications.

C The chest radiograph typically shows a fine ground glass appearance.

D The chest radiograph typically shows patchy consolidation with areas of hyperinflation.

E The chest radiograph typically shows unilateral abnormalities.

A

D The chest radiograph typically shows patchy consolidation with areas of hyperinflation.

Meconium aspiration syndrome has a mortality of 25%. Pneumothorax and pneumomediastinum are commonly seen.

61
Q

QUESTION 68
A 3-year-old boy presents with a short history of shortness of breath. Clinical examination is unremarkable, but on the chest radiograph there are multiple pulmonary nodules suggestive of metastases. Which one of the following tumours would be the most likely source of pulmonary metastases in this child?

A Lymphoma

B Medulloblastoma

C Nephroblastoma (Wilms’ tumour)

D Neuroblastoma

E Testicular teratoma

A

C Nephroblastoma (Wilms’ tumour)

Both testicular and Wilms’ tumours metastasise to lungs; however, the age of the patient is highly atypical for a testicular tumour (more common in adolescence and young adulthood).

62
Q

QUESTION 69
A 4-year-old child presents with shortness of breath and a fever. The chest radiograph shows a round opacity within the right lower zone. No previous radiographs are available for comparison. Which one of the following statements is true when trying to distinguish pneumonia from a tumour in a child?

A Ill-defined margins make pneumonia more likely.

B Pneumonic changes usually persist for several weeks following treatment.

C Sharp margins are associated with a round pneumonia.

D The absence of an air bronchogram makes tumour more likely.

E An MRI would be the next investigation of choice.

A

A Ill-defined margins make pneumonia more likely.

Round pneumonia tends to have ill-defined margins, lacks an air bronchogram and resolves quickly with treatment. Correlative history is crucial

63
Q

QUESTION 76
A 12-year-old child with CF has been followed up with annual chest radiographs. Which of the following features is a late radiographic change associated with the disease?

A Bronchial wall thickening

B Cavitations

C Consolidation

D Diffuse interstitial pattern

E Hilar enlargement

A

E Hilar enlargement

Hilar enlargement results from enlarged pulmonary vasculature and is a sign of pulmonary hypertension.

64
Q

QUESTION 77
A neonate with a history of worsening cyanosis and respiratory distress has a series of chest radiographs taken. The initial chest radiograph reveals a solid left upper lobe mass and over the course of 3 weeks, this becomes aerated. Progressive mediastinal shift is seen as the mass enlarges. Which one of the following is the most likely diagnosis?

A Bronchopulmonary sequestration

B Congenital cystic adenomatoid malformation (CCAM)

C Congenital diaphragmatic hernia

D Congenital lobar emphysema

E Pneumatocoele secondary to E. coli infection

A

D Congenital lobar emphysema

CCAM may present in a similar manner, but as the mass becomes aerated it appears as a multicystic, fluid-filled mass.

65
Q
  1. You are reviewing the daily radiograph on a 4-week-old neonate in the neonatal ICU. This patient was born at 32 weeks and was diagnosed with uncorrected transposition of the great arteries with an intact ventricular septum for which he underwent an emergency balloon atrial septostomy for palliation, whilst awaiting an arterial switch operation (ASO). As part of your routine practice, you review the position of the lines. The ET tube is located 9 mm from the carina. The right internal jugular vein central line is sited in the mediastinum, 8 mm inferior to the carina. The umbilical venous catheter (UVC) passes superiorly from the umbilicus, with its tip at the inferior aspect of the right atrium. The umbilical arterial line (UAC) passes inferiorly before passing superiorly, with its tip located at the level of T9. The nasogastric tube tip is below the diaphragm. Which of these pieces of apparatus may be incorrectly sited?

A. ET tube.

B. Nasogastric tube.

C. UAC.

D. Central line.

E. UVC.

A
  1. D. Central line.

The tip of the central line is ideally situated superior to the level of the carina to ensure that it is not within the right atrium. Whilst placement inferior to this may be satisfactory, it can result in placement within the right atrium or coronary sinus, which are associated with an increased risk of complications. When assessing the ET tube in neonates, the tip should lie between the carina and the thoracic inlet, as more accurate placement is difficult given the size of the patient. UACs are divided into high and low lines. High lines should have their tip between T8 and T10. Low lines should be sited below L3. Lines should not be sited between T10 and L3 as they can cause thrombosis of the mesenteric or renal vessels. UVCs pass superiorly into the portal vein, along the ductus venosus into the IVC.

66
Q
  1. A male neonate born at 38 weeks gestation develops acute respiratory distress within 36 hours of delivery. Clinical examination reveals coarse breath sounds. Serial CXRs carried out in the Special Care Baby Unit (SCBU) reveal reduced lung volumes and widespread granular opacities. There is reduced transradiancy in the right hemithorax and an ultrasound reveals that this is secondary to a mild-moderate pleural effusion. What is the most likely diagnosis?

A. Surfactant deficiency.

B. Meconium aspiration.

C. Bronchopulmonary dysplasia (BPD).

D. Beta haemolytic streptococcal pneumonia.

E. Pulmonary interstitial emphysema (PIE).

A
  1. D. Beta-haemolytic streptococcal pneumonia.

This is the most common neonatal pneumonia and differs from other common patterns of pneumonia as it causes this pattern on CXR, whereas other neonatal pneumonias commonly cause high lung volumes and streaky perihilar densities. Transient tachypnoea of the newborn (TTN) and meconium aspiration syndrome also classically cause increased lung volumes and streaky perihilar density, although the radiographic appearance of meconium aspiration can also often be difficult to differentiate from beta-haemolytic streptococcal pneumonia. A useful differentiation is that pleural effusions are uncommon in meconium aspiration, but are commonly seen in beta-haemolytic streptococcal pneumonia. TTN is also characterized by rapid clearance and would not be expected to persist on serial radiographs. The other common cause for low lung volumes and granular densities described is surfactant deficiency. This would be very unlikely in an infant born at 38 weeks gestation.

67
Q
  1. A male neonate born at 26 weeks gestation is currently being treated in your neonatal ICU. The patient’s mother received corticosteroids prior to delivery and prophylactic surfactant administration as per your department’s standard practice. The CXR was clear for the first 7 days. Despite this the child developed streaky perihilar granular opacities and respiratory difficulties. Further surfactant administration has been carried out, but the CXR carried out today (day 28 postpartum) shows small streaky linear densities along with cystic bubbly lucencies, which have been becoming increasingly prominent over the last 7 days and are distributed in an irregular pattern bilaterally. What is the most likely explanation for this appearance?

A. Surfactant deficiency.

B. Meconium aspiration.

C. BPD.

D. Beta haemolytic streptococcal pneumonia.

E. PIE.

A
  1. C. BPD.

Whilst surfactant deficiency is undoubtedly a feature of this case, the evolution of the clinical scenario indicates that a further condition is evolving to explain the findings and clinical condition. In this case the two likeliest conditions are BPD and PIE, both most commonly associated with immature lungs and both of which give bubbly lucencies on radiography. PIE is a feature of air leak phenomena which occur in stiff lungs and is due to either high airway pressure or alveolar overdistention causing passage of gas into the interstitial spaces. It is associated with other airleak phenomena such as pneumopericardium. BPD was originally described to occur in four stages, but the advent of refined ventilation, surfactant, and prophylactic administration of corticosteroids, have changed the typical progression. A complete discussion of these diseases is found in the article referenced below. BPD tends to develop more gradually than PIE (as described in the clinical vignette) and tends to occur later than PIE.

68
Q
  1. A 3-month-old child presents to the paediatric outpatient clinic with a history of recurrent respiratory distress. The child had an uneventful delivery, but has had recurrent problems since birth. The child had a CXR taken prior to discharge home, aged 2 days, which showed a density in the left upper lobe, felt by the paediatrician to represent the thymus. Whilst the infant has never required admission, the mother is concerned due to recurrent coughing and dyspnoea. A CXR obtained at the clinic shows a large hyperlucent area in the left upper lobe. What is the most likely diagnosis?

A. Congenital lobar emphysema.

B. Congenital cystic adenomatoid malformation.

C. Pulmonary sequestration.

D. Persistent PIE.

E. Congenital diaphragmatic hernia.

A
  1. A. Congenital lobar emphysema.

This has a lobar predilection with around 40% being found in the left upper lobe. Congenital lobar emphysema initially presents as an area of soft tissue density due to retained foetal pulmonary fluid. This resolves and is replaced by hyperlucency. Most present in the neonatal period with respiratory distress, but they can present later.

Congenital cystic adenomatoid malformations (CCAM) do not show a lobar predilection, but can be found anywhere. They can be either air or fluid filled and consist of multiple cysts.

CCAM are graded on the size of the cysts with type 1 lesions containing one or more large cysts, type 2 have numerous small cysts, and type 3 contain microscopic cysts, but appear solid at imaging.

Congenital diaphragmatic hernias are also initially solid on plain radiography and only contain air if there is bowel present within the hernia and this contains air. This would obviously have continuity with the diaphragm and not be contained entirely within the upper lobe.

Persistent PIE occurs when PIE fails to resolve after 1 week. As PIE is almost always seen in infants with surfactant deficiency being ventilated, it would not be in the differential in this case. Sequestrations are usually solid masseson plain film radiography, unless there has been a history of infection within the sequestration.

69
Q
  1. One of the obstetricians in your hospital refers a pregnant patient, who is at 34 weeks gestation, to your department for assessment, as an antenatal ultrasound has shown a mass in the chest of the foetus. A repeat ultrasound does confirm a hyperechoic mass confined to the inferior aspect of the left hemithorax. This mass is vascular. Incidental note is also made of a ventricular septal defect. An MRI confirms the presence of a high-signal intrathoracic mass in the left lower thorax, which derives its blood supply from the abdominal aorta. The lung surrounding this area appears unremarkable. What is the most likely diagnosis?

A. Congenital diaphragmatic hernia.

B. Intralobar sequestration.

C. Congenital cystic adenomatoid malformation.

D. Foregut duplication cyst.

E. Extralobar sequestration.

A
  1. E. Extralobar sequestration.

Sequestrations are defined as lung tissue masses with no normal connection to the bronchial tree or pulmonary arteries.

This diagnosis can often be difficult to make as extralobar sequestrations are associated with CCAM and congenital diaphragmatic hernia (CDH).

The fact that the lesion is contained entirely within the thorax, with no abdominal connection described, means CDH is less likely.

CCAM usually derive their blood supply from the pulmonary arteries.

Foregut duplication cysts would be expected to be hypoechoic on ultrasound and non-vascular.

The differentiation between intralobar and extralobar sequestrations is more difficult.

Previously the venous drainage has been used, with intralobar described as draining to the pulmonary veins and extralobar to systemic veins (IVC or azygous commonly). In actuality, the drainage is variable and the pleural covering the only constant difference, with intralobar having a separate visceral pleural covering, but extralobar having completely separate pleura.

Extralobar sequestrations are more common in males and are usually in the left hemithorax (90%). They are also more commonly associated with other congenital abnormalities, such as cardiac malformations.

70
Q
  1. A CT chest has been requested for a neonate in the neonatal ICU. This infant was born at 27 weeks gestation and developed right-sided PIE during the first week of life. The neonatologists practiced selective left bronchial intubation and no further air leak sequelae occurred. Also present on the CXR is a hyperlucent lesion in the right lower lobe. This is not clearly seen on the initial radiographs due to the generalized haziness present due to the surfactant deficiency. This lesion is not increasing in size and is not causing any significant respiratory embarrassment, but requires further assessment to define treatment. On CT a focal lesion is present confined to the right lower lobe, which consists of multiple cystic structures with central linear densities. This area demonstrates mild expansion. What is the diagnosis?

A. Congenital cystic adenomatoid malformation.

B. Persistent PIE.

C. Congenital diaphragmatic hernia.

D. Congenital lobar emphysema.

E. Bronchogenic cyst.

A
  1. B. Persistent PIE.

Although alluded to in the clinical scenario, this should not be assumed to be the most likely diagnosis in the absence of the CT findings, as this is an extremely uncommon condition. The CT findings provide the diagnosis due to the linear densities within the cystic cavities representing the bronchopulmonary bundle surrounded by air within the interstitial space. This appearance is seen in over 80% of cases. The abnormality is often confined to a single lobe, but can be more widespread. Current optimal management is debated. Lesions increasing in size are thought to be best treated with surgical resection, with stable lesions often resolving over time with conservative management.

71
Q

@# (Chest) 56. An 18-year-old woman with Poland syndrome is being assessed by plasticsurgery for reconstruction. As part of her pre-operative work-up a CT chest is requested. What is the classic finding in this disorder?

A. Absence of the sternal head of pectoralis major.

B. Hypoplastic clavicles.

C. Anterior protrusion of the ribs.

D. Bilateral breast aplasia.

E. Anterior protrusion of the sternum.

A
  1. A. Absence of the sternal head of pectoralis major.

Poland syndrome is an uncommon congenital unilateral chest wall deformity characterized by partial or total absence of the greater pectoral muscle and ipsilateral syndactyly.

Associated anomalies include ipsilateral breast aplasia and atrophy of the second to fifth ribs.

Hypoplastic clavicles are a feature of cleidocranial dysostosis.

Anterior protrusion of the ribs gives rise to pectus excavatum, whereas anterior protrusion of the sternum is seen in pectus carinatum.

72
Q
  1. A 6-year-old girl is brought to your local paediatric outpatients with a history of night sweats, tiredness, and new onset wheeze not responding to bronchodilators. A CXR is done which shows increased mediastinal soft tissue noted superiorly. The paravertebral lines are maintained. The aortic knuckle is not visible. A lateral CXR has been carried out at the request of the paediatrician, which shows increased soft tissue displacing the trachea posteriorly, causing mild narrowing. What is the most likely diagnosis?

A. Tuberculosis.

B. Lymphangioma.

C. Bronchogenic cyst.

D. Thymic/nodal malignant infiltration.

E. Teratoma.

A
  1. D. Thymic/nodal malignant infiltration.

The first step in this question is to localize the lesion. The posterior displacement of the trachea indicates an anterior mediastinal position. It is then necessary to consider the common causes of anterior mediastinal masses in children, which are: normal thymus and thymic/nodal infiltration (leukaemia, lymphoma), with other causes (lymphangioma and teratoma) being much less common. A normal thymus would not be expected to cause significant posterior displacement of the trachea. On plain film it would not be possible to differentiate thymic infiltration from anterior mediastinal nodal infiltration. TB in children would be uncommon in the anterior mediastinum, especially when no abnormality is noted in the hila.

73
Q

@# 8 An infant is born at 41 weeks following a traumatic delivery with passage of meconium. He quickly develops respiratory distress with grunting, tachypnoea and nasal flaring. A CXR is performed. What is the most likely appearance of the CXR?

a Widespread ground-glass opacities

b `White out’ of the lung

C Widespread patchy consolidation and air trapping

d Focal consolidation with air bronchograms

e Small volume lungs with bilateral pleural effusions

A

8 Answer C: Widespread patchy consolidation and air trapping

Meconium aspiration is the most common cause of neonatal respiratory distress in full-term or postterm infants. Severe hypoxaemia induces a gasping reflex with resulting inhalation of meconium, which causes medium and small airway obstruction and chemical pneumonitis.

74
Q

@# 4 A premature infant, born at 27 weeks, was ventilated for the first 14 days and subsequently on CPAP for a further 21 days. A CXR is performed at three months of age. What is the likely appearance?

a `White out’ of the lungs

b Patchy bilateral ground-glass opacities with air bronchograms

c Hyperinflation with coarse linear densities and focal areas of emphysema

d Reduced lung volumes with bilateral reticulonodular opacities

e Normal volume lungs with no focal abnormality

A

4 Answer C: Hyperinflation with course linear densities and focal areas of emphysema

The infant described has bronchopulmonary dysplasia, which is caused by oxygen toxicity and barotrauma in infants on assisted ventilation at >21 % oxygen for >28 days. Appearances change with time:

75
Q

16 A 15-year-old girl with a history of recurrent lower respiratory tract infections during early childhood has a CXR to investigate a cough. The lungs are asymmetrical with a smaller, hyperlucent left lung with a small left hilum and evidence of air trapping during expiration. Which of the following is the most likely diagnosis?

a Congenital cystic adenomatoid malformation

b Congenital lobar emphysema

C Swyer James syndrome

d Pulmonary artery atresia

e Intralobar pulmonary sequestration

A

16 Answer C: Swyer-James syndrome

Swyer-James syndrome is a chronic complication of viral bronchiolitis with recurrent infections preventing normal development of the lung. HRCT appearances include areas of normal lung attenuation within hypoattenuating lung, air trapping and bronchiectasis

76
Q

35 A 12-year-old boy develops progressive shortness of breath on exercise and coughing at night. He has had two chest infections in the last year but his CXR is normal. What is the most likely diagnosis?

a Cystic fibrosis

b Asthma

C Bronchiectasis

d Congenital lobar emphysema

e Tracheo-oesophageal fistula

A

35 Answer B: Asthma

The CXR is often normal in the early stages of asthma with abnormalities developing with increasing severity (e.g. hyperinflation, bronchiectasis, scars from recurrent infections).

77
Q

38 A three-year-old girl with a chronic cough and recurrent chest infections is found to have atelectasis, mucous plugging, cystic bronchiectasis, and air trapping on CXR. What is the likely diagnosis?

a Congenital lobar emphysema

b Whooping cough

C Cystic fibrosis

d Asthma

e Cystic adenomatoid malformation

A

38 Answer C: Cystic fibrosis

Cystic fibrosis is an autosomal-recessive disease characterised by mucous plugging of exocrine glands secondary due to exocrine gland dysfunction producing thick obstructing mucus and reduced mucociliary clearance

78
Q

(Chest) 39 A 20-year-old male presented to the Emergency Department with chest pain. A chest radiograph demonstrated an abnormal density behind the heart and while his symptoms resolved rapidly the abnormality persisted on repeat imaging. A CT showed it to be a mass within the left lower lobe with a systemic arterial supply. What additional finding would make a diagnosis of intralobar bronchopulmonary sequestration more likely than an extralobar type?

a The presence of multiple small systemic feeding arteries

b An associated diaphragmatic hernia

c A more solid appearance to the mass

d Separate pleural layer around the mass

e Drainage via pulmonary veins

A

39 Answer E: Drainage via pulmonary veins

The other findings are all more typical of an extralobar sequestration.

79
Q

49 A two-year-old boy is brought into the Emergency Department by his mother because she is concerned he has aspirated a foreign body. No foreign body is seen on CXR. What other CXR features might suggest recent foreign body aspiration?

a Hyperlucency

b Consolidation

C Effusion

d Pneumothorax

e Bronchiectasis

A

49 Answer A: Hyperlucency

Foreign bodies are usually non-radioopaque. Many present with hyperlucency, caused by air trapping and hyperinflation distal to the foreign body. Chronic foreign body aspiration can present as atelectasis, recurrent pneumonia, or bronchiectasis. CT is the most sensitive method to detect obstruction.

80
Q

53 A 15-year-old boy is found to have metastatic deposits in his lungs. Osteogenic sarcoma has been excluded. What is the most likely primary tumour?

a Rhabdomyosarcoma

b Wilms’ tumour

C Ewing’s sarcoma

d Medulloblastoma

e Retinoblastoma

A

53 Answer C: Ewing’s sarcoma

The four paediatric tumours that tend to metastasise to the lungs are:
* rhabdomyosarcoma primarily affects children ages 1-5 and 15-19
* osteogenic sarcoma primarily affects children under age 15
* Wilms’ tumour primarily affects females under five years of age with a high incidence in African Americans.?

81
Q

67 An infant is brought to the Emergency Department with a cough and temperature. The CXR reveals a well-circumscribed, rounded mass lesion behind the heart with air bronchograms. Which of the following would be appropriate further imaging?

a Enhanced CT chest

b Lateral chest radiograph

c Ultrasound chest

d Repeat CXR in six weeks

e High resolution CT chest

A

67 Answer D: Repeat CXR in six weeks

The infant has a round pneumonia, most commonly caused by Haernophilus influenzae and Streptococcus. Although uncomplicated pneumonia in children does not require follow-up imaging, those patients with round pneumonia, pneumatocele or pulmonary abscess should have a repeat frontal CXR in six weeks to confirm resolution.

82
Q

70 A neonate born at term develops respiratory distress within the first few hours after birth. He is noted to have a scaphoid abdomen. A CXR is performed that shows multiple cystic structures within the left hemithorax. Which of the following is the most likely diagnosis?

a Bochdalek hernia

b Morgagni hernia

c Bronchogenic cyst

d Congenital lobar emphysema

e Intralobar pulmonary sequestration

A

70 Answer A: Bochdalek hernia

Bochdalek hernia is a posterolateral defect in the diaphragm. It presents in the neonatal period and is more commonly (80%) on left. Morgagni hernia rarely presents clinically at birth and is anteromedial congenital diaphragmatic hernia.

83
Q

8 A three-year-old girl with a cough and fever has a CXR. This shows a spherical mass in the posterior left lower lobe with slightly ill-defined borders and air bronchograms. Which of the following is the most likely diagnosis?

a Aspergilloma

b Round pneumonia

C Pertussis

d Bronchiolitis

e Neuroblastoma metastasis

A

8 Answer B: Round pneumonia

Round pneumonia is seen in children. The most common causative organisms are Haernophilus influenzae, Streptococcus and pneurnococcus.

84
Q

12 A CXR is performed as part of a septic screen on a lethargic two year old who does not have any specific chest signs. The lungs and pleural spaces appear clear, but a large anterior mediastinal mass is identified, the edge of which appears wavy. Which of the following is the likely cause?

a Reactive lymphadenopathy

b Normal thymus

C Non-Hodgkin’s lymphoma

d Thymic hyperplasia

e Hodgkin’s lymphoma

A

12 Answer B: Normal thymus

The normal thymus can be very large and have extreme asymmetry. The edge is often wavy due to anterior rib end impressions. The thymus appears largest at about two years old, although it continues to grow into adolescence.

85
Q

16 A neonate born at term is admitted to the neonatal intensive care unit with respiratory distress and generalised hypotonia. A CXR is performed that does not show any focal abnormality, but the chest is noted to be bell-shaped. Which of the following is the likely underlying diagnosis?

a Down’s syndrome

b Group B streptococcal pneumonia

c Foetal alcohol syndrome

d Congenital toxoplasmosis

e Neuromuscular disease

A

16 Answer E: Neuromuscular disease

The normal newborn chest is broader than the adult with near parallel lateral margins.

86
Q

20 A three month old with feeding difficulties and intermittent respiratory distress has a CXR. This shows a well-defined homogeneous triangular mass adjacent to the posterior medial hemidiaphragm. A contrast-enhanced CT chest is performed and the mass seen on the CXR corresponds to a homogeneous well-circumscribed soft-tissue mass, which drains into the right heart via the IVC. Which of the following is the most likely diagnosis?

a Congenital cystic adenomatoid malformation

b Intralobar pulmonary sequestration

c Extralobar pulmonary sequestration

d Neuroblastoma

e Morgagni diaphragmatic hernia

A

20 Answer C: Extralobar pulmonary sequestration

This is an accessory lobe with its own pleural sheath, which prevents collateral air drift, resulting in an airless round mass. Both intralobar and extralobar pulmonary sequestrations have systemic arterial supplies.

87
Q

(CNS) 21 The report on a child’s neck X-ray reads, `There is loss of the normal subglottic angle resulting in a wine bottle-shaped appearance on AP projection. On the lateral film there is an ill-defined haziness of the soft-tissue/air interface between the glottic and subglottic regions.’ What is the most likely diagnosis?

a Croup

b Supraglottitis

c Congenital tracheal stenosis

d Subglottic haemangioma

e Laryngomalacia

A

21 Answer A: Croup

These are the classical XR findings in croup. In supraglottitis one would expect to see the `thumb sign’ of a short, broad epiglottis.

Plain films in congenital tracheal stenosis would reveal a narrowed tracheal lumen.

In subglottic haemangioma, indicative findings are a soft-tissue mass in the subglottic region extending inferiorly, narrowing the airway.

Laryngomalacia is a clinical and endoscopic diagnosis, but findings consistent with laryngomalacia are anterior bowing and inferior displacement of the aryepiglottic folds.

Films taken during expiration would show abnormal persistent dilatation of the oropharyngeal airway, which is indicative of laryngomalacia.

88
Q

24 A three-day-old infant presents with poor feeding and lethargy. He was born at term following prolonged rupture of membranes. A CXR shows bilateral patchy infiltrates and a left-sided pleural effusion. Which of the following is the most likely diagnosis?

a Respiratory distress syndrome

b Group B Streptococcal pneumonia

C Meconium aspiration

d Congenital listeriosis

e Staphylococcal pneumonia

A

24 Answer B: Group B Streptococcal pneumonia

Group B Streptococcus is the most common cause of neonatal pneumonia. Pleural effusions are seen in two-thirds of cases. Risk factors include prolonged rupture of membranes, maternal fever during labour and preterm labour.

89
Q

28 An infant born at 26 weeks requires intubation and ventilation for respiratory distress. A CXR is performed at eight hours of age. What is the likely appearance of the CXR?

a Left lower lobe consolidation and bilateral pleural effusions

b Hyperinflation with areas of air trapping

C Bilateral patchy ground-glass opacities with air bronchograms

d Bilateral basal atelectasis

e Bilateral patchy consolidation with no evidence of air bronchograms

A

28 Answer C: Bilateral patchy ground-glass opacities with air bronchograms

Respiratory distress syndrome, or hyaline membrane disease, is caused by surfactant deficiency due to immature type II alveolar cells. Premature infants, multiple gestations, perinatal asphyxia and infants of diabetic mothers are at risk. The onset of symptoms is up to five hours after birth.

90
Q

32 An 11-year-old girl with recurrent sinusitis and a chronic productive cough has a CXR. This shows bronchial wall thickening and cystic spaces with airfluid levels. What additional findings would be expected?

a The heart lying in the normal position with the stomach bubble on the right

b The heart and stomach bubble lying in the normal position

C The left ventricle lies on the right and the stomach bubble lies on the left

d The left ventricle lies on the right with the stomach bubble on the right

e The heart is of normal shape but is displaced to the right

A

32 Answer D: The left ventricle lies on the right and the stomach bubble lies on the right

The findings are those of Kartagener’s syndrome. This is a familial disorder characterised by situs inversus, nasal polyposis with chronic sinusitis and bronchiectasis.

91
Q

(CVS) 39 A four-year-old male presented with a chronic cough. A chest radiograph demonstrated hyperinflation of the lungs with peribronchial thickening and he was treated with a course of antibiotics. Follow-up radiographs demonstrated persistent changes and the development of ring shadows in the right upper lobe. A CT revealed bronchiectasis bilaterally in the upper lobes and bronchoalveolar lavage samples grew Staphylococcus aureus and Pseudomonas aeruginosa. What is the most likely underlying diagnosis?

a Asthma

b Aspergillosis

C Cystic fibrosis

d Post-infective bronchiectasis

e Tuberculosis

A

39 Answer C: Cystic fibrosis

Hyperinflation and peribronchial thickening are non-specific signs and could be seen in asthma or aspergillosis, but the further imaging findings make asthma unlikely and the organisms grown make aspergillosis and TB less likely. Patients with cystic fibrosis are more susceptible to pseudomonas infections.

92
Q

52 A child presents with chronic cough, shortness of breath, excess sputum production, recurrent chest infections, and haemoptysis. His CXR shows bronchial wall thickening. What other features would you expect to see to confirm your diagnosis?

a Dilated cystic spaces with air-fluid levels

b Consolidation

c Pulmonary infiltrates

d Soft-tissue mass

e Pleural effusion

A

52 Answer A: Dilated cystic spaces with air-fluid levels

This child has bronchiectasis, which is an irreversible dilatation of bronchi often with bronchial wall thickening. In severe cases a honeycombing pattern may be seen.

93
Q

54 A six-year-old is investigated for malaise, weight loss and shortness of breath. A CXR shows multiple, well-defined spherical opacities of varying sizes throughout both lungs. Which of the following is the most likely underlying diagnosis?

a Testicular carcinoma

b Medulloblastoma

C Retinoblastoma

d Rhabdomyosarcoma

e Brainstem glioma

A

54 Answer D: Rhabdomyosarcoma

Lung metastases in children most commonly come from rhabdomyosarcoma, osteosarcoma, Wilms’ tumour and Ewing’s sarcoma.

94
Q

@# 59 A neonate has a CXR for respiratory distress following a precipitous normal vaginal delivery at term. This shows mild cardiomegaly and mild hyperexpansion but no focal lung abnormality. Which of the following is the most likely underlying diagnosis?

a Respiratory distress syndrome

b Transient tachypnoea of the newborn

C Meconium aspiration

d Group B Streptococcal pneumonia

e Pulmonary haemorrhage

A

59 Answer B: Transient tachypnoea of the newborn

Cardiomegaly in neonates has multiple causes including hypoglycaemia, congenital heart disease, asphyxia and infants of diabetic mothers.

95
Q

63 A 12-year-old girl has developed a chesty cough and temperature. Her Emergency Department CXR shows some interstitial infiltrates and multifocal lobar consolidation. Which of the following is the most likely pathogen?

a Adenovirus

b Chlarnydia trachornatis

c Bordetella pertussis

d RSV

e Mycoplasma pneurnoniae

A

63 Answer E: Mycoplasma pneunaoniae

Mycoplasma pneurnoniae is the commonest cause of pneumonia in children of school age. There is a spectrum of CXR findings from interstitial infiltrates through to dense consolidation. It is often multifocal. Small effusions are seen in 20% and hilar adenopathy may be seen.

Bordetella pertussis causes whooping cough in incompletely immunised or very young infants. CXR findings include a `shaggy heart’ caused by central patchy infiltrates and bronchial wall thickening.

Chlarnydia trachornatis causes a barking cough and is associated with conjunctivitis in infants (6-12 weeks of age) due to transvaginal inoculation at birth.

Adenovirus is the most frequent viral pneumonia; CXR findings include adenopathy, peribronchial thickening, hyperinflation, and scattered bilateral subsegmental infiltrates.

RSV (respiratory syncytial virus) is the most common cause of lower respiratory infection in children under two years of age.

96
Q

5 The CXR of a tachypnoeic term neonate delivered by Caesarean section shows linear densities radiating from the hila, thick fissures and small pleural effusions. What is the most likely diagnosis?

a Transient tachypnoea of the newborn

b Hyaline membrane disease

C Meconium aspiration

d Bronchopulmonary dysplasia

e Pulmonary haemorrhage

A

5 Answer A: Transient tachypnoea of the newborn

Transient tachypnoea of the newborn is caused by inadequate clearance of the lung fluid before the first breath. It occurs typically in infants born by Caesarean section or precipitous delivery.

97
Q

12 A four-year-old girl is referred by her GP with a six-month history of persistent cough and wheeze despite being treated for several chest infections. She has a CXR, which reveals a cystic mediastinal mass, with widening of the carina and mild narrowing of the proximal aspect of both left and right main bronchi. The lung volumes are normal. What is the most likely diagnosis?

a Lymphoma

b Hiatus hernia

c Tracheo-oesophageal fistula

d Pericardiac tumour

e Bronchogenic cyst

A

12 Answer E: Bronchogenic cyst

Bronchogenic cysts are relatively rare but are the commonest cystic lesion of the mediastinum causing partial obstruction of the trachea or bronchus, which may lead to emphysema.

Eighty-five per cent of bronchogenic cysts occur in the mediastinum; 15% are intrapulmonary.

98
Q

22 A five-year-old girl complains of shortness of breath and on CXR is found to have an enlarged cardiac silhouette with discrete calcific densities. Ultrasound reveals a pericardial complex cystic mass containing calcific foci and a pericardial effusion. What are the calcific foci most likely to be?

a Bone

b Teeth

C Stones

d Foreign bodies

e Calcified pericardial cyst

A

22 Answer B: Teeth

This child has a pericardial teratoma, which is a benign germ cell tumour. It can cause respiratory distress and cyanosis due to pericardial tamponade and compression of the SVC, right atrium, aortic root, and pulmonary artery. Calcified pericardial cysts are extremely rare in children and usually asymptomatic.

99
Q

33 A two-year-old boy is brought in with a sudden onset of cough. On CXR there is a radio-opaque midline opacity and atelectasis. Which of the following additional features are you most likely to see?

a Consolidation

b Cardiomegaly

C Pneumothorax

d Pulmonary infiltrates

e Air trapping and hyperinflation

A

33 Answer E: Air trapping and hyperinflation

This child has aspirated a foreign body, which often causes air trapping and hyperinflation, and less often gives rise to pulmonary infiltrates.

100
Q

(Chest) 34 An 18-year-old female with recurrent chest infections and left-sided chest pain was investigated and found to have an 8-cm smooth oval mass in the left lower zone on her chest radiograph. Further investigation with CT shows multiple air-filled thin-walled cysts, which did not communicate with the bronchial tree. The mass enhances at the same time as the thoracic aorta. Where is the venous drainage of the mass most likely to lead?

a Pulmonary veins

b Azygos vein

c Superior vena cava

d Intercostal veins

e Coronary veins

A

34 Answer A: Pulmonary veins

Bronchopulmonary sequestration is a congenital malformation of a non-functioning lung segment that has no communication with the tracheobronchial tree and has a systemic blood supply.

In intralobar sequestration, which is more common, aeration may occur via the pores of Kohn and venous drainage is via pulmonary veins to the left atrium.

Venous drainage is via systemic veins in extralobar sequestration.

101
Q

36 An 11-year-old boy presents with abdominal pain, nausea and vomiting and has an inhaler for a chronic cough. His CXR reveals gas-filled loops of bowel in the centre of his chest. What is the likely diagnosis?

a Bochdalek hernia

b Morgagni hernia

C Septum transversum defect

d Hiatal hernia

e Eventration

A

36 Answer B: Morgagni hernia

This boy has a Morgagni hernia, which is an anteromedial parasternal defect caused by the maldevelopment of the septum pellucidum and tends to present in older children. Bochdalek hernias are posterolateral defects and tend to present in babies. The chronic cough may have been misdiagnosed as asthma but may well be related to the hernia.

102
Q

(Chest) 38 A 20-year-old man presented with his fifth episode of pneumonia in two years. A chest radiograph revealed an abnormal density behind the left cardiac shadow and consolidation distal to this lesion. A CT was performed, which showed a soft-tissue mass with a feeding vessel from the thoracic aorta. What is the most likely diagnosis?

a Bronchopulmonary sequestration

b Bronchogenic cyst

C Cystic adenomatoid malformation

d An anomalous left pulmonary artery

e Arteriovenous malformation

A

38 Answer A: Bronchopulmonary sequestration

This is a congenital anomaly resulting from independent development of part of the tracheobronchial tree, which may have a systemic arterial blood supply and classed into two types - intra and extralobar. The intralobar type is more common and classically presents in a young adult as recurrent pneumonia. They have a pulmonary venous drainage and are contained within the normal visceral pleura. An extralobar sequestration has its own visceral pleura, drains into systemic veins and is usually asymptomatic.

103
Q

40 The newborn daughter of a diabetic mother is noted to have nasal flaring and expiratory grunting. The CXR shows complete lung white out. What is the likely diagnosis?

a Respiratory distress syndrome

b Neonatal pneumonia

C Unilateral pulmonary agenesis

d Cystic fibrosis

e Persistent foetal circulation syndrome

A

40 Answer A: Respiratory distress syndrome

This baby has respiratory distress syndrome, which is an acute pulmonary disorder characterised by generalised atelectasis, intrapulmonary shunting, ventilation-perfusion abnormalities and reduced lung compliance due to surfactant deficiency. Term infants of diabetic mothers are predisposed to the condition.

104
Q

44 A four-month-old boy presents with cough and difficulty feeding. On examination he is in respiratory distress with nasal flaring and subchondral recession and there is widespread wheeze. A CXR shows hyperexpansion and prominent hila with no focal parenchymal abnormality. Which of the following is the most likely diagnosis?

a Respiratory syncytial virus bronchiolitis

b Streptococcal pneumonia

c Varicella pneumonitis

d Haemophilus pneumonia

e Croup

A

44 Answer A: Respiratory syncytial virus bronchiolitis

Respiratory syncytial virus (RSV) causes 75% of cases of bronchiolitis, most commonly affecting infants between three and six months old.

105
Q

47 A CXR of a newborn is shown to have an indistinct right heart border with opacification in this area. A few days later the CXR shows a well-defined lucency in the right hemithorax with displacement of the mediastinum to the left, widened rib interspaces, and an intact right hemidiaphragm. What is the most likely diagnosis?

a Venolobar syndrome

b Sequestration

C Congenital lobar emphysema

d Cystic adenomatoid malformation

e Lymphangiectasia

A

47 Answer C: Congenital lobar emphysema

This newborn has congenital lobar emphysema of the middle lobe. Congenital lobar emphysema mainly presents in the neonatal period and most commonly affects the left upper lobe, then the middle lobe, followed by the right upper lobe. Initially, there is opacity due to slow clearance of foetal lung fluid, which clears over the first days of life, then hyperinflates.

106
Q

(CVS) 47 A 30-year-old male who had a history of recurrent respiratory infections as a child presented with exertional dyspnoea. On his chest radiograph there is increased transradiancy of the right lung. What additional feature would support a diagnosis of Swyer-James syndrome?

a Bronchiectasis

b Mismatched ventilation and perfusion defects on V/Q scan with delayed washout in hyperlucent areas

C An increase in size of the ipsilateral pulmonary vessels

d Small contralateral hilum

e A right-sided aortic knuckle

A

47 Answer A: Bronchiectasis

As a late complication of viral bronchiolitis in childhood, bronchiectasis is the most correct answer. On V/Q scan a matched defect is seen in the affected areas, and the ipsilateral vessels and hilar are small.

107
Q

52 A young boy with tracheal stenosis undergoes a CT of his chest. He is found to have a tracheal bronchus. What findings would you expect to see?

a Right upper lobe apical segmental bronchus originates from the trachea rather than the right upper lobe bronchus

b Left upper lobe apical segment originates from the trachea rather than the left upper lobe bronchus

C Right lower lobe apical segment bronchus originates from the trachea rather than the right lower lobe bronchus

d Left lower lobe apical segment bronchus originates from the trachea rather than the left lower lobe bronchus

e Right upper lobe anterior segmental bronchus originates from the trachea rather than the right upper lobe bronchus

A

52 Answer A: Right upper lobe apical segmental bronchus originates from the trachea rather than the right upper lobe bronchus

This is also known as a `pig bronchus’ since in a pig the right upper lobe bronchus normally arises from the trachea. There is also an association with tracheal stenosis. Rarely, there may be a duplicated bronchus with a normally located right upper lobe apical segmental bronchus present as well.

108
Q

55 A three-year-old girl is brought into hospital with vomiting, having swallowed household cleaning fluid. Her CXR at 10 hours is normal. How should this be interpreted?

a She is likely to have aspirated but the findings have not yet appeared on CXR

b She is likely to have aspirated but the findings have resolved

c She is unlikely to have aspirated

d The CXR is most likely to be inadequate

e She can be discharged home

A

55 Answer C: She is unlikely to have aspirated

Hydrocarbon aspiration can occur from emesis post ingestion of gasoline, kerosene, household cleaners and polishes. CXR findings are usually significant at two to eight hours after ingestion, which includes pulmonary infiltrates due to pneumonitis. Pneumatoceles may develop as sequelae.

109
Q

@# 58 A CT chest is performed on a six year old. An incidental soft-tissue mass is seen in the middle mediastinum. Which of the following is it most likely to represent?

a Neurogenic tumour

b Great vessel aneurysm

c Thymoma

d Lateral meningocele

e Teratoma

A

58 Answer B: Great vessel aneurysm

Mediastinal lesions by location:
* anywhere in the mediastinum - lymphoma/leukaemia, adenopathy, mediastinitis, haematoma
* anterior mediastinum - teratoma, thyroid, thymoma
* middle mediastinum - bronchopulmonary foregut malformations, hiatal hernia/other gastric or oesophageal abnormality, cardiac/pericardiac tumours and cysts, great vessels aneurysms/anomalies
* posterior mediastinum - neurogenic tumours, neurenteric cysts, lateral meningoceles, spinal tumours/ osteomyelitis/ discitis, descending aortic or azygous anomaly/aneurysm, extramedullary haematopoiesis.

110
Q

60 A 16-year-old girl is having a third CT scan of her chest, which reveals dilated and beaded bronchi, bronchial wall thickening, and peripheral bronchial mucous plugging. There is also hyperinflation and hilar enlargement, which has progressed since the previous scan. What is the most likely diagnosis?

a Bronchiectasis

b Cystic fibrosis

C Swyer James syndrome

d juvenile rheumatoid arthritis

e Systemic lupus erythematosus

A

60 Answer B: Cystic fibrosis

In cystic fibrosis the lungs are initially normal but abnormal chloride secretions result in viscoid airway mucus, which predisposes to infection, inflammatory response and airway obstruction of small and large airways. There is progressive bronchiectasis and chronic obstruction, resulting in hyperinflation. Adenopathy may develop causing hilar enlargement and later pulmonary hypertension with dilated central pulmonary arteries may be seen. Increasing cardiac size indicates cor pulmonale and poor survival without transplantation. Pneumomediastinum and pneumothorax can also occur.

111
Q

@# 62 A two year old with pneumonia is not improving with antibiotics. CXR shows hyperinflation and patchy infiltrates. What is the likely diagnosis?

a Chlarydia

b Streptococcus pneuroniae

c Pneurnocystis carinii

d RSV

e Adenovirus

A

62 Answer D: RSV

The commonest organisms causing pneumonia at different ages:
* Premature infants - (1) Group B Streptococcus, (2) E. coli, (3) Listeria, (4) CMV
* Infants - (1) RSV, (2) Chlarnydia, (3) Streptococcus pneumoniae, (4) Haernophilus influenzae type B
* School age - (1) Mycoplasma, (2) Influenza A, (3) Streptococcus pneurnoniae.

112
Q

67 A 15-year-old boy has a small, deformed right chest wall. The CXR demonstrates that the soft tissues over the right hemithorax are thinned. On CT there is an absence of the right-sided pectoral muscles. What is likely diagnosis?

a Askin tumour

b Poland syndrome

C Pectus excavatum

d Pectus carinatum

e Muscular dystrophy

A

67 Answer B: Poland syndrome

Poland syndrome is a congenital defect with unilateral underdevelopment or absence of pectoralis muscles. There is usually ipsilateral syndactyly. The right side is more commonly affected. In pectus excavatum there is a depressed sternum with left-sided displacement of the heart and loss of definition of the right heart border (resembling right middle lobe pneumonia) on CXR. In pectus carinatum there is a `pigeon breast’ with prominence of the sternum. Askin tumour is an aggressive neuroectodermal chest wall tumour, which is destructive and invasive.

113
Q

(CVS) 67 A female neonate developed respiratory distress and after investigation was found to have congenital lobar emphysema. Which part of her lung was most likely to have been affected?

a Right upper lobe

b Right middle

C Right lower lobe

d Left upper lobe

e Left lower lobe

A

67 Answer D: Left upper lobe

A hyperlucent expanded lobe is seen (which may initially appear of soft tissue density due to retained fluid). The condition preferentially affects the left upper lobe (43 %), right middle lobe (32 %) and right upper lobe (20%). Two lobes are affected in 5%. Treatment is resection of the affected lobe.

114
Q
  1. An 8-year-old chid presents with high fever and left side chest pain. The chest radiograph demonstrates a 3 cm round lesion in left lower zone with ill-defined margins. No air bronchograms seen. The most likely diagnosis is?

(a) Mycoplasma infection

(b) Tuberculosis

(c) Round pneumonia

(d) Solitary lung metastasis

(e) Congenital cystic adenomatoid malformation

A
  1. (c) Round pneumonia

Typical features of round pneumonia are seen in the history. This resolves rapidly on treatment.
Mycoplasma infections give diffuse reticulonodular shadowing or segmental consolidation. Congenital cystic adenomatoid malformation presents with dyspnea with radiographs showing cystic/large bulla lesions in lung. Metastases usually have well-defined margins.

115
Q
  1. A sick neonate has an anteroposterior radiograph of the chest and abdomen. It shows an umbilical catheter line traversing initially caudally and then cephalad, and the tip lies to the left of T3 vertebral body. The catheter is in?

(a) Correctly placed umbilical arterial line

(b) Correctly placed umbilical vein line

(c) High umbilical arterial line

(d) High umbilical vein line

(e) Low umbilical arterial line

A
  1. (c) A high umbilical arterial line

The umbilical arterial line passes caudad into the internal and common iliac arteries and then courses cephalad in the aorta. The tip should be above the level of celiac axis (T6–T10), or below the renal arteries (L3–L5). An umbilical vein catheter courses directly cephalad on the right side. The tip should lie above the liver and not passed into a tributary vein.

116
Q
  1. A premature baby of a diabetic mother delivered by caesarean section develops tachypnoea soon after birth. Chest radiographs show hyperinflated lungs with prominent interstitial markings and prominent horizontal fissure. These changes resolved after 3 days. The most likely diagnosis is?

(a) Respiratory distress syndrome

(b) Meconium aspiration syndrome

(c) Transient tachypnoea of the newborn

(d) Left heart failure

(e) Normal lung of newborn

A
  1. (c) Transient tachypnoea of newborn

If the processes of clearing amniotic fluid from the lungs is impaired in a new born transient tachypnoea of the newborn develops. This is associated with prematurity, caesarean section and diabetic mothers. These are typical radiographic features, which resolve in 2–3 days.

117
Q
  1. A premature baby with hypoxia was treated with mechanical positive pressure ventilation. Subsequent radiographs show worsening appearances with hyperexpansion of the left lung, mediastinal shift to the right and appearance of small bubbles radiating from the hilum. The most likely diagnosis is?

(a) Pulmonary interstitial emphysema

(b) Respiratory distress syndrome

(c) Transient tachypnoea of newborn

(d) Cystic fibrosis

(e) Congenital lobar emphysema

A
  1. (a) Pulmonary interstitial emphysema

This condition is typically associated with premature babies treated with mechanical ventilation. Most commonly, the air may leak from the parenchyma leading to pneumothorax. Air may also leak into the interstitial space and spread throughout the lymphatics and along the perivascular sheaths causing interstitial emphysema.

118
Q

(Chest) 24. An 18-year-old man presents with a history of recurrent chest infections. The chest radiograph shows a left paraspinal soft tissue density behind the heart. Contrastenhanced CT shows a 6 cm lobulated, multicystic lesion in the left lower lobe containing solid and cystic components. There is a feeding artery from the thoracicaorta into the lesion. What is the most likely diagnosis?

(a) Extralobar sequestration

(b) Intralobar sequestration

(c) Congenital cystic adenomatoid malformation

(d) Abscess

(e) Lipoid pneumonia

A
  1. (b) Intralobar sequestration

These are the typical features of an intralobar sequestration. Extralobar sequestration often is seen usually before 6 months age and associated with other congenital anomalies. The other causes listed do not have any feeding vessels.

119
Q
  1. A 1-day-old neonate presents with respiratory distress. The chest radiograph shows soft tissue shadowing in the right lower zone. On day 4, CT of the chest shows multiple small cysts of varying sizes containing air with resolution of the soft tissue density. What is the most likely diagnosis?

(a) Bronchopulmonary sequestration

(b) Congenital diaphragmatic hernia

(c) Pneumonia

(d) Congenital cystic adenomatoid malformation

(e) Bronchogenic cyst

A
  1. (d) Congenital cystic adenomatoid malformation

These are multicystic lesions filled with air. They communicate with the bronchial tree and are filled with air early in life. Most lesions are confined to a single lobe and are solitary. Sequestration does not contain air in the neonatal period and is only filled with air if infected.

120
Q
  1. A neonate presents with respiratory distress. The chest radiograph shows a hyperinflated right lung with flattened right hemidiaphragm and deviation of the mediastinum to the left. What is the most likely diagnosis?

(a) Congenital lobar emphysema

(b) Congenital cystic adenomatoid malformation

(c) Pulmonary hypoplasia

(d) Pneumothorax

(e) Congenital diaphragmatic hernia

A
  1. (a) Congenital lobar emphysema

Progressive overdistention of a pulmonary lobe due to obstruction. Initially, after birth the lobe may be filled with fetal lung fluid and then gradually fluid is replaced by air. The lung hyperexpands and causes mediastinal shift and flattening of the diaphragm. CT shows attenuation of pulmonary vessels as compared to the opposite side.

121
Q
  1. Which of the following statements are correct?

(a) The normal thymus in a child is hypoechoic relative to the liver.

(b) The thymus is highly vascular on color Doppler ultrasound.

(c) The thymus arises from the third and fourth branchial pouches.

(d) Teratomas comprise the most common anterior mediastinal mass in childhood.

(e) Thymolipomas are common causes of thymic enlargement in childhood.

A

Answers:
(a) Correct
(b) Not correct
(c) Correct
(d) Not correct
(e) Not correct

Explanation:
Normal thymus is hypovascular on color Doppler. Thymic hyperplasia is the most common anterior mediastinal mass in childhood. It may be secondary to hyperthyroidism, myasthenia gravis and rebound growth following illness or stress. Thymomas and Thymolipomas are extremely rare in childhood. Neoplastic involvement is usually secondary to infiltration by leukemia or lymphoma.

122
Q
  1. Which of the following are correct regarding bronchopulmonary sequestration: (T/F)

(a) Intralobar sequestration (IS) typically presents in the neonatal period.

(b) Extralobar sequestration (ES) is more common than intralobar sequestration.

(c) The main blood supply is form bronchial arteries.

(d) Most commonly affect the lower lobes.

(e) Can cause recurrent chest infection in adults.

A

Answers:
(a) Not correct
(b) Not correct
(c) Not correct
(d) Correct
(e) Correct

Explanation:
Bronchopulmonary sequestration is a non-functioning sequestered lung segment which has no communication with the tracheobronchial tree and has a systemic blood supply.

Supply is commonly from a separate branch from the aorta and sometimes from upper abdominal vessels or coronary arteries.

IS is more common (80%) and ES is found in (20%).

IS typically presents in adulthood and is often an incidental finding.

123
Q
  1. Which of the following are true regarding round pneumonia? (T/F)

(a) It is most commonly associated with Klebsiella infection.

(b) It occurs most commonly in the second and third decades of life.

(c) It is more common in the upper lobes.

(d) It frequently progresses to cavitation.

(e) It is a feature of Q-fever infection.

A

Answers:
(a) Not correct
(b) Not correct.
(c) Not correct.
(d) Not correct.
(e) Correct.

Explanation:
It is most commonly associated with Streptococcus pneumonia.

Round pneumonia occurs most frequently in children within first decade of life.

It is usually seen in lower lobes, often abutting pleural space.

Round pneumonia evolves rapidly over a few days into segmental consolidation (sometimes with air bronchograms).

Cavitation is unusual

124
Q
  1. Which of the following are correct regarding congenital diaphragmatic hernias (T/F)

(a) Most congenital hernias are of the Morgagni type

(b) Defective closure of the pleuroperitoneal membranes leads to a Bochdalek hernia

(c) Right sided hernias may have a delayed presentation

(d) Bochdalek hernias are usually left sided

(e) Congenital cystic adenomatoid malformation is a differential diagnosis

A

Answers:
(a) Not correct
(b) Correct
(c) Correct
(d) Correct
(e) Correct

Explanation:
Bochdalek hernia is most common congenital hernia (85%-90%). Morgagni type is seen in 10%-15% of the cases.

125
Q
  1. Which of the following are correct regarding congenital lobar emphysema (T/F)

(a) It commonly affects the lower lobes

(b) Bilateral involvement is rare

(c) It typically presents in the perinatal period.

(d) Underlying vascular markings are present

(e) The affected lobe is opaque after birth

A

Answers:
(a) Not correct
(b) Correct
(c) Not correct
(d) Correct
(e) Correct

Explanation:
Congenital lobar emphysema most commonly affects left upper lobe. Lower lobes involvement is seen in only 2 %. In 25% cases, presentation is not seen in perinatal period.

126
Q
  1. Features of Swyer-James syndrome include: (T/F)

(a) Increased lucency of a hemithorax

(b) Increased hilar markings, with peripheral pruning

(c) Mild bronchiectasis

(d) Air trapping during expiration

(e) Typically a lobar or segmental distribution

A

Answers:
(a) Correct
(b) Not correct
(c) Correct
(d) Correct
(e) Not correct

Explanation:
Swyer-James syndrome is a rare lung condition that manifests as unilateral hemithorax lucency as a result of postinfectious obliterative bronchiolitis. It is generally characterized on radiographs by a unilateral small lung with hyperlucency and air trapping. Diminished vascular markings are seen.

127
Q
  1. Which of the following are correct regarding pulmonary sequestration:

(a) 70-80 % of cases area intra-lobar

(b) Usually communicates with the tracheobronchial tree.

(c) The majority present in the first 6 months of life.

(d) Blood supply is from the pulmonary arteries

(e) The posterior part of the left lower lobe is most frequently involved

A

Answers:
(a) Correct
(b) Not correct
(c) Not correct
(d) Not correct
(e) Correct

Explanation:
Pulmonary sequestration is a congenital abnormality consisting of non-functioning primitive lung tissue not communicating with tracheobronchial tree. Intralobular form is more common and presents in adulthood with recurrent chest infections and high output cardiac failure. The blood supply is systemic, mostly from descending thoracic aorta. Contrast enhancement at the same time as thoracic aorta is characteristic on CT.

128
Q
  1. The following are correct regarding umbilical catheters in neonates: (T/F)

(a) Typically, there are two umbilical veins and one umbilical artery.

(b) Arterial catheters initially go caudally and posteriorly before coursing cephalad.

(c) Venous catheters follow an anterior and cephalad course

(d) The tip of a venous catheter should be placed below the right atrium

(e) Portal vein gas is a bad prognostic sign

A

Answers:
(a) Not correct
(b) Correct
(c) Correct
(d) Correct
(e) Not correct

Explanation:
Typically there are two umbilical arteries and one umbilical vein. Air may be introduced inadvertently in intrahepatic portal venous system at time of umbilical venous catheter insertion. This is usually transient.